Задачи по математике на смекалку 6 класс с ответами: Логические задачи с ответами, задания для детей на логику и смекалку
By: Date: 09.11.1970 Categories: Разное

Содержание

Логические задачи с ответами, задания для детей на логику и смекалку


Классические логические задачи


Вопросы, загадки, задачи на логику, смекалку и сообразительность — хороший набор для развития
пытливости детского ума, любознательности и интереса к учебе, для полезного семейного досуга.


Регулярные тренировки в решении задач на логику помогают ребенку развивать нестандартное
мышление. Текстовые логические задачи, задания на поиск закономерностей, выстраивание
последовательностей особенно рекомендованы дошколятам и младшим школьникам.


Категории задач по возрасту с ответами и комментариями

Смотрите примеры задач на развитие логического мышления из Лаборатории LogicLike и
решайте онлайн.


Интересные нестандартные задачи на логику


Занимательные сюжеты, привлекательные для детей картинки, обучающие подсказки и комментарии к
ответам.


7 логических задач для разминки

Логические задачи, задачи на логику. С ответами.

Размер

Толщина

Фон

 Скрыть 
 ответы 
 +шрифт 

 —шрифт 
 +жирн 

 —жирн 

White
Cyan
LGreen
GYellw
DpSkBl
Coral
DPink1
DPink2
SkBlue
Orange
OlivD1
OlivD2
LBlue
PGreen
Yellow
Gold
Blue
Green
Wheat
Chocol
Salmon
Red
HPink
DPink
Pink
VioRed
Magent
Violet
Plum
Purple
OrRed
Bisque
Bisqu2
LemC1
LemC2
Corns2
Honey2
Turqu1
Turqu2
SGrn1
SGrn2
Orchi1
Orang1
Gray
golrod



Как вы думаете, если полста разделить на половину, то сколько в итоге получится?

Ответ

Получится не 25, как многие могут подумать, а 100. Так как если 50 разделить на 1/2, то это равносильно умножению на 2.



Если три десятка умножить на четыре десятка, то сколько получится?

Ответ

Получится не 12 десятков, а 120 десятков. То есть : 30 * 40 = 1200.



Можете ли вы обосновать, почему почти во всех странах мира канализационные крышки у люков имеют только круглую форму? (Квадратные крышки люков бывают лишь тогда, когда они дополнительно крепятся шарнирами).

Ответ

Если крышки люков будут квадратными, то они могут легко провалиться в люк, т.к. диагональ квадрата больше стороны квадрата. Поэтому их если и делают, то только прикрепив к люку шарнирами. У круглых крышек люков нет диагонали и стороны, а только диаметр, который у крышки всегда больше отверстия люка.



Как вы думаете, какой знак следует поставить между 0 и 1, чтобы было получено число больше 0, но меньше 1?

Ответ

Этот знак является запятой. То есть 0,1. Это число больше 0, но меньше 1.



Как вы думаете, сколько граней имеет шестигранный карандаш, который ни разу не затачивали?

Ответ

Шестигранный карандаш, если не подвергался заточке будет иметь 8 граней. 6 большие грани и 2 торцевые.



Трехлитровый сосуд полностью заполнен тремя литрами воды. Вам необходимо за 2 переливания заполнить два пустых сосуда на 1 и 2 литра, чтобы в каждом из них было по 1 литру воды. При этом больше нельзя пользоваться ни чем, кроме этих трех сосудов.

Ответ

Из полного сосуда наливаем в двухлитровый пустой ровно два литра, т.е. до краев. Далее из этого сосуда выливаем в однолитровый ровно литр воды (т.е. до краев).



Как вы думаете, существуют ли линии отличные от окружности, на которых все точки будут равноудалены от какой-то одной точки?

Ответ

Равноудаленностью всех точек обладает любая линя, лежащая на поверхности шара.



Как вы думаете, какой предмет будет иметь одинаковое изображение при рисовании его с любой точки зрения?

Ответ

Этим свойством обладает только шар.



Попробуйте сообразить, какой из выводов, указанных ниже, верный :

А) Здесь три ложных вывода.

Б) Здесь один ложный вывод.

В) Здесь два ложных вывода.

Г) Здесь пять ложных выводов.

Д) Здесь четыре ложных вывода.

Ответ

Правильный вариант Д — здесь четыре ложных вывода. В связи с тем, что один является верным, а остальные не верные.



Попробуйте догадаться сколько стоит книга, если книга стоит доллар плюс пол книги.

Ответ

Книга стоит 2 доллара. Решение : полкниги стоит доллар, значит вся книга стоит 2 доллара.


Поделитесь с друзьями:


Ответьте, сколько сейчас времени, если оставшаяся часть суток в два раза превышает прошедшую?

Ответ

Сейчас восемь часов.



Некий бизнесмен захотел привезти в Японию для продажи 10 000 пар первоклассных дорогих кроссовок. Но в Японии на такие кроссовки накладываются очень большие пошлины. Подумайте и скажите, как же хитроумный бизнесмен смог ввезти все эти кроссовки в Японию, при этом заплатил только очень небольшие деньги? (Никакой коррупционной и преступной составляющей здесь нет).

Ответ

Бизнесмен поступил очень хитро. Он разделил каждую пару кроссовок и отправил весь объем двумя партиями. То есть в одной партии были только кроссовки на левую ногу, во второй только на правую ногу. Одну партию он отправил в Токио, другую в Осака. В каждом из городов бизнесмен не заплатил пошлину и товары были конфискованы и выставлены на аукционе. В связи с тем, что никому не была нужна партия кроссовок только на одну ногу, то бизнесмен выкупил сам обе партии за мизерные деньги.



5 рыбаков съели 5 карпов за 5 дней. Как вы думаете, а за сколько дней 15 рыбаков съедят 15 карпов?

Ответ

15 рыбаков съедят 15 карпов тоже за 5 дней. Если 5 рыбаков съедают 5 карпов за определенный промежуток времени, то у 15 рыбаков скорость поедания карпов в 3 раза больше, следовательно за 5 дней они съедят 15 карпов.



В мешке имеется 9 кг сахара. Есть также и две гири по 50г и 200г. Подумайте, как за три взвешивания на чашечных весах отвесить 2кг сахара?

Ответ

Сперва необходимо на чашечных весах разделить содержимое мешка пополам на 4,5кг в каждой чашке. Далее одну чашу опустошаем, и снова 4,5кг делим пополам и получаем в каждой чаше весов по 2,25кг. В третье взвешивание уже нужно опустошить обе чаши, но из одной чаши 2,25кг сахара положить в отдельный мешок. И далее при помощи гирек в 200г и 50г (итого 250г) отвесить из пакета с 2,25кг ровно 250г. Тогда в пакете останется ровно 2 кг.



Два колхозника решили узнать, у кого больше овец. Первый из них сказал : «если ты дашь мне свою козу, то у меня будет их в два раза больше, чем у тебя». Второй ему говорит : «А давай лучше ты мне дашь свою одну овцу, тогда у меня овец будет столько же, сколько и у тебя». Сколько же овец у каждого из колхозников? (Передачи овец пока еще не было).

Ответ

У первого колхозника 7 овец, у второго только 5. Если первый колхозник отдает одну овцу второму и их становится поровну, то значит, что изначально у первого их на 2 больше. Если же второй колхозник отдает овцу первому, то их становится у первого в 2 раза больше, такое возможно, только если у первого изначально было 7 овец, а у второго 5.



В одном классе всего 36 учеников. Девочек на 3 больше, чем мальчиков. Сколько мальчиков и девочек в этом классе?

Ответ

Если разделить 36 пополам, то получим 18, т.е. две половины класса по 18 человек. Если из первой половины добавить школьника в другую, то получится разница в 2 человека. Если отнять еще одного и добавить снова в большую часть, то получим превышение на 4 человека. Следовательно задача не имеет решения.



Можете ли вы записать число 1000 при помощи только восьми восьмерок и арифметических знаков суммы?

Ответ

Получится равенство : 888 + 88 + 8 + 8 + 8 = 1000.



На столе лежат 4 монеты, из которых одна сделана из другого металла и отличается по весу, хотя внешне они все одинаковые. Как определить эту монету за 2 взвешивания на чашечных весах?

Ответ

Варианты взвешиваний : 1) ложем на весы 1 и 2 монеты, если они равны по весу, то одну монету заменяем на третью. Далее если они равны, то отличная монета 4-я, если не равны, то 3-я монета отличная от остальных. 2) ложем на весы 1 и 2 монеты, если они не равны по весу, то вместо одной монеты ложем 3-ю. Если уравновешиваются, то отличная убранная монета, если не уравновешиваются, то отличная от других монет оставшаяся на весах старая монета.



Как так могло оказаться, что половина числа 12 стало равно 7 ?

Ответ

Нужно написать число 12 римскими цифрами : IIX , далее провести посередине линию. Верхняя половина будет в виде VII, что соответствует цифре 7.



На праздничном столе горят 7 свечей. 3 из них потушили. Сколько свечей останется?

Ответ

Останутся 3 потушенные свечи, т.к. остальные 4 сгорят полностью.


Поделитесь с друзьями:

10 логических задач для нестандартного мышления / Newtonew: новости сетевого образования

Логические задачи — пожалуй, самый эффективный инструмент для развития логики и мышления как у детей, так и у взрослых.

Решение задачи на логику предполагает сложный мыслительный процесс. Это последовательное совершение определённых логических действий, работа с понятиями, использование различных логических конструкций, построение цепочки точных рассуждений с правильными промежуточными и итоговыми умозаключениями.

В отличие от большинства математических и других видов задач, при решении логических задач ключевым является не нахождение количественных характеристик объекта, а определение и анализ отношений между всеми объектами задачи.

Используйте комплексный подход

Среди всего многообразия логических задач часто дети выбирают себе пару любимых категорий и погружаются в их решение. Достаточно ли этого?

Наверняка большинство из нас хотя бы раз проходили тесты на уровень логики. Большинство их составлено из одних силлогизмов или вопросов с подвохом. Мы не предлагаем подобные тесты, потому что точно знаем, что определить уровень развития логического мышления с помощью десятка или двух вопросов, даже приблизительно, невозможно. Так же, как и развить нестандартное мышление, решая только отдельные типы логических задач.

Классические логические, комбинаторные и истинностные задачи, закономерности и математические ребусы, задачи про фигуры в пространстве и развертки, на перестановки и движение, на взвешивание и переливание; решаемые с конца, с помощью таблиц, отрезков, графов или кругов Эйлера – это далеко не все разнообразие логических задач, при решении которых активизируются всевозможные мыслительные операции и развивается творческое, нестандартное мышление.

Логика — это вкусняшка для ума

Именно так написали на доске ученики перед началом одного из занятий нашего кружка по логике. В чём же прелесть логических задач?

  • они будут одинаково интересны и увлечённым математикой детям, и «гуманитариям»;
  • многие из них не требуют знаний школьной программы;
  • их может решать даже дошкольник без навыков чтения (например, судоку, ребусы, головоломки со спичками, «шестерёнки» и другие задачи в картинках).

Дети любят решать логические задачи и загадки. Им это интересно! Когда я работала в школе, я видела, что ребята справляются с программой, механически запоминая способ решения тех или иных типовых задач.

А задачи со звёздочками сразу оживляли класс, в процесс обсуждения включались и сильные, и слабые ученики. Дома эту задачу дети уже могли и хотели сами объяснить родителям. Но даже эти задачи со звёздочками были расположены на страницах учебника случайным образом, не было выработано никакой системы.

 

Битно Галина Михайловна

завуч LogicLike, учитель высшей категории

Только системный и комплексный подход создаёт благоприятные предпосылки для формирования нестандартного мышления. «Пища для ума» тоже должна быть сбалансированной и разнообразной. Попробуйте сами и предложите вашим детям решить именно такую подборку задач. Это поможет выявить те звенья в логике, над которыми стоит поработать усерднее.

Попробуйте сами

В онлайн-платформе Logiclike, созданной для развития логики и математических способностей у детей 5-12 лет, авторы постарались реализовать всё то, чего зачастую так не хватает и ученикам, и учителям в школьных программах. Системность, вовлечение, интерактивность, наглядность, мотивация… Но первым делом это — пища для ума, та самая «вкусняшка», которая заставляет ребенка думать, рассуждать, проверять свои силы, проявлять творческий подход и радоваться, когда удаётся найти правильное решение.

Рекомендации от методистов и учителей LogicLike:

  • Хотите развить у ребенка нестандартное мышление и гибкую логику – давайте ему хорошую зарядку для ума в виде разнообразных логических задач, для решения которых нужно использовать разные логические законы и методы решения (метод с конца, табличный метод, с помощью графов или кругов Эйлера и т.д.)
  • Подходите к обучению системно: от теории к задачам, от простого к сложному, от знакомства с новыми типами заданий к рефлексии.
  • Учитывайте специфику мышления у детей младшего школьного возраста – используйте визуальные образы и наглядные материалы.
  • Важно не навязывать детям способ решения, а стараться проводить разбор так, чтобы они сами путем логических рассуждений нашли правильный ответ.
  • Внедряйте игровые элементы в процесс обучения, используйте обучающие возможности IT.
  • Занятия логикой, как и спортивные тренировки, нуждаются в регулярности и постепенном повышении сложности задач.

Занимайтесь вместе с ребенком и с удовольствием!

 

27 января 2017, 12:00

Мнение автора может не совпадать с позицией редакции.

Скопировать ссылку

АВТОРСКАЯ КОЛОНКА

ЛогикЛайк

LogicLike.com — образовательная онлайн-платформа для детей 5-12 лет, их родителей, а также любознательных взрослых. Мы рассказываем, как тренировать мышление и математические способности, публикуем логические задачи и тесты, делимся мыслями об образовании.

Нашли опечатку? Выделите фрагмент и нажмите Ctrl+Enter.

Логические задачи для тренировки — Школа 52, Владивосток

Страница 1 из 2

Задачи для подготовки к самостоятельной работе.

 

1.  Министры иностранных дел России, США и Китая обсудили за закрытыми дверями проекты соглашения о полном разоружении, представленные каждой из стран. Отвечая затем на вопрос журналистов: «Чей именно проект был принят?», министры дали такие ответы:

Россия — «Проект не наш, проект не США»;
США — «Проект не России, проект Китая»;
Китай — «Проект не наш, проект России».
Один из них (самый откровенный) оба раза говорил правду; второй (самый скрытный) оба раза говорил неправду, третий (осторожный) один раз сказал правду, а другой раз — неправду.

Определите, представителями каких стран являются откровенный, скрытный и осторожный министры.

2.  В 8, 9 и 10-ом классах учатся три товарища: Наиль, Зиннур и Альберт. Один из них занимается в кружке информатики, другой -радиолюбитель, третий – авиамоделист. Скажите, в каком классе учится и в каком кружке занимается каждый из них, если известно, что:

8-й класс посетил лесхоз, 9-й – кондитерскую фабрику,
10-й класс — автозавод;
во время посещения лесхоза Зиннур опасался, как бы не разбить лежавшую в кармане радиолампу;
Наиль ушёл на стадион один, так как его товарищ был занят налаживанием своей модели самолёта;
4) товарищ авиамоделиста очень заинтересовался конвейером на автозаводе.

3. Однажды в международном лагере отдыха за круглым столом оказалось пятеро парней из Москвы, Санкт-Петербурга, Новгорода, Казани и Уфы. Их имена: Саша, Никита, Руслан, Петя и Миша.

Москвич сидел между уфимцем и Мишей, санкт-петербужец – между Сашей и Никитой, а напротив него сидели казанец и Руслан. Петя никогда не был в Санкт-Петербурге, а Саша не бывал в Москве и Уфе. Уфимец с Никитой регулярно переписываются.
В каком городе живёт каждый из ребят?

4. Лауреатом женского поэтического конкурса стали пять поэтесс.
1-е место заняла мисс Мартин. Бетти Мун не писала стихотворение «Сорока». Своё стихотворение Мэри сочинила в мае. «Муза» была написана в сентябре. Джастина Моран написала «Зрелый день». Стихотворение «Человек» принадлежит перу Кэт. «Муссон» был создан Сюзан в марте. Мисс Мегге написала своё стихотворение в апреле. Фамилия Мэри не Мульдон, а одно из стихотворений написано в октябре.
Назовите имя и фамилию каждой поэтессы, название стихотворения, которое она написала, а также время его создания.

5. Три свидетеля дорожного происшествия сообщили сведения о скрывшемся нарушителе. Боб утверждает, что тот был на синем «Рено». Джон сказал, что нарушитель ехал на черной «Тойоте», а Сэм сказал, что машина была точно не синяя, и, по всей видимости, это был «Форд». Когда удалось отыскать машину, выяснилось, что каждый из свидетелей точно определил только  один из параметров автомобиля, а в другом ошибся. Машина какой марки и какого цвета была у нарушителя?

6. Внимание Андрея, Дениса и Марата привлек промчавшийся мимо них автомобиль.
— Это английская машина марки «Феррари» — сказал Андрей.
— Нет, машина итальянская марки «Понтиак», — возразил Денис.
— Это «Сааб», и сделан он не в Англии, — сказал Марат.
Оказавшийся рядом знаток автомобилей сказал, что каждый из них прав только в одном из двух высказанных предположений.
Какой же марки этот автомобиль и в какой стране изготовлен?

7. Трое друзей, болельщиков автогонок «Формула-1», спорили о результате предстоящего этапа гонок.
— Вот увидишь, Шумахер не придет первым, — сказал Джон. — Первым будет Хилл.
— Да нет же, победителем будет, как всегда, Шумахер! — воскликнул Ник. — А об Алези и говорить нечего, ему не быть первым.
Питер, к которому обратился Ник, возмутился:
— Хиллу не видать первого места, а вот Алези пилотирует самую мощную машину.
По завершении этапа гонок оказалось, что каждое из двух предположений двоих друзей подтвердилось, а оба предположения третьего из друзей оказались неверны. Кто выиграл этап гонки?

8 Три дочери писательницы Дорис Кей — Джуди, Айрис и Линда тоже очень талантливы. Они приобрели известность в разных видах искусств — пении, балете и кино. Все они живут в разных городах, поэтому Дорис часто звонит им в Париж, Рим и Чикаго, Известно, что:
1) Джуди живет не в Париже, а Линда — не в Риме;
2) Парижанка не снимается в кино;
З) Та, кто живет в Риме, певица;
4) Линда равнодушна к балету.
Где живет Айрис и какова ее профессия?

9 В симфонический оркестр приняли на работу трех музыкантов — Брауна, Смита и Вессона, умеющих играть на скрипке, флейте, альте, кларнете, гобое и трубе. Известно, что:

1) Смит — самый высокий;
2) играющий на скрипке меньше ростом играющего на флейте;
З) играющие на скрипке и флейте и Браун любят пиццу;
4) когда между альтистом и трубачом возникает ссора, Смит мирит их;
5) Браун не умеет играть ни на трубе, ни на гобое.
На каких инструментах играет каждый из музыкантов, если каждый владеет двумя инструментами?

10. После традиционного вечера встречи с бывшими выпускника­ми школы в стенгазете появилась заметка о трех наших бывших учениках. В этой заметке было написано, что Иван, Андрей и Борис стали учителями. Теперь они преподают разные дисциплины: один — математику, второй — физику, а тре­тий — химию. Живут они тоже в разных городах: Минске, Витебске и Харькове. В заметке было еще написано, что первоначальные их планы осуществились не полностью: Иван работает не в Минске, Андрей — не в Витебске; житель Минска преподает не математику, Андрей преподает не физику. Повезло только жителю Витебска: он преподает любимую им химию. Кто где живет и что преподает?

11. В педагогическом институте Казакова, Андреева, Покатина, Галеев, Шакиров и Веселов преподают философию, математику, английский язык, французский язык, немецкий язык, историю. 

Преподаватель немецкого языка и преподаватель математики в студенческие годы занимались художественной гимнастикой.
Шакиров старше Веселова, но стаж работы у него меньше, чем у преподавателя философии.
Будучи студентками, Казакова и Андреева учились вместе в одном университете. Все остальные окончили педагогический институт.
Веселов – отец преподавателя французского языка.
Преподаватель английского языка – самый старший из всех по возрасту и по стажу работы. Он работает в этом институте с тех пор, как окончил его. Преподаватели математики и истории – его бывшие студенты.
Казакова старше преподавателя немецкого языка.
Кто какой предмет преподаёт?

12. Одиннадцать ребят: Александр, Борис, Василий, Георгий, Дмитрий, Евгений, Захар, Иван, Кирилл, Леонид и Михаил – учатся все в разных классах одной школы.
Старший брат Дмитрия оканчивает 7-й класс, а младший брат Евгения учится в 5-ом классе. Александр старше Кирилла на один класс, Леонид старше Евгения на два класса, а самый старший из мальчиков Михаил. Борис помогает в учёбе Евгению, Дмитрий – Ивану, Георгий – Александру.
Иван при окончании 4-го класса получил похвальную грамоту.
3) Борис – вожатый в 5-ом классе, а Василий – в 4-ом классе.
4) Александр, Кирилл и шестиклассник занимаются в гимнастической секции, а одновременно с ними тренируются баскетболисты, среди которых всегда Борис, Евгений и восьмиклассник.
5) Александр и семиклассник живут на улице Лесной, Георгий и пятиклассник – на улице Красивой, Дмитрий, первоклассник и восьмиклассник – на Садовой, а Кирилл и десятиклассник – на Солнечной.
Кто из них в каком классе учится?

13. Три одноклассника — Влад, Тимур и Юра, встретились спустя 10 лет после окончания школы.
Выяснилось, что один из них стал врачом, другой физиком, а третий юристом.
Один полюбил туризм, другой бег, страсть третьего — регби.
Юра сказал, что на туризм ему не хватает времени, хотя его сестра — единственный врач в семье, заядлый турист.
Врач сказал, что он разделяет увлечение коллеги.
Забавно, но у двоих из друзей в названиях их профессий и увлечений не встречается ни одна буква их имен.
Определите, кто чем любит заниматься в свободное время и у кого какая профессия.

 

 

ГДЗ по Математике за 5‐6 класс Задачи на смекалку МГУ

Математика 5 класс
Шарыгин И.Ф.
задачи на смекалку

Авторы: Шарыгин И.Ф.

«Решебник по Математике 5‐6 класс Задачи на смекалку Шарыгин (Просвещение)» сделает трудный и требовательный предмет «математика» элементарным и любимым для каждого пятиклассника и шестиклассника. Техническая наука требует полной самоотдачи, а уроки по ней проходят практически ежедневно. Это налагает свой отпечаток на манеру обучения, нагрузка велика с самого начала, и лишь увеличивается с каждым годом. Чтобы справиться с полным объёмом материала, рекомендуется заручиться поддержкой онлайн-решебника. Тогда у школьника исчезнут переживания насчёт неудовлетворительных оценок по предмету – в дневнике напротив графы «математика» будут только пятерки и четверки.

Подробнее о достоинствах ГДЗ по математике 5-6 класс задачи на смекалку Шарыгин

Решебник имеет массу полезных свойств:

  • он доступен онлайн, обладает мобильной версией, пользоваться можно с любого современного гаджета «под рукой»;
  • верные ответы есть на все номера вопросов и упражнений из дидактического дополнительного пособия;
  • молодые люди становятся более независимыми и уверенными в себе после работы вместе с ГДЗ, им больше не понадобится помощь пап и мам, репетитора;
  • решебник обладает нативно понятной навигацией;
  • ученик может проверить свою работу по сборнику и исправить все недочеты, опередив преподавателя;
  • лучше запоминается материал после самостоятельного контроля ошибок.

Онлайн-ресурс станет палочкой-выручалочкой для ребят в пятом и шестом классе, дети смогут решить пример любой сложности, найти ответ на самый «заковыристый» вопрос из учебника. В эти года ребята узнают:

  1. Как найти наибольший общий делитель и наименьшее кратное.
  2. Какое свойство дроби является основным в математике.
  3. Каким образом применить распределительное свойство умножения.
  4. Знакомство с некоторыми определениями шара и его частей, радиуса, диаметра, связь между ними.
  5. Как выполнять упрощение выражений.
  6. Изучение формулы для нахождения площади прямоугольника.

Чтобы освоение дисциплины проходило наиболее эффективно, подготавливать домашние задания удастся наиболее продуктивно под руководством пособия «Решебник по Математике 5‐6 класс Задачи на смекалку Шарыгин И. Ф. (Просвещение)». Благодаря онлайн-ГДЗ любое упражнение из дидактического дополнения к учебнику не доставит трудностей учащемуся.

Упс! Какое-то из ваших приложений или расширений браузера ломает код сайта. Пожалуйста, выключите их и перезагрузите страницу. Почините gdz-putina.fun!

Реально сложные задачи — Журнал «Код»: программирование без снобизма

В отличие от предыдущей задачи, здесь решение намного сложнее, потому что в голове нужно держать одновременно 2-3 условия, которыми надо проверять числа. Но мы справимся.

Для решения нам понадобится вспомнить, что такое простые числа и в чём их особенность. Простое число — то, которое может делиться нацело только на себя и на единицу. Например, число 5 — простое, потому что делится только на 5 и на 1. А число 6 — не простое, потому что кроме 6 и 1 оно ещё делится на 2 и 3 без остатка. Семь тоже будет простым числом, а восемь — нет, потому что кроме 8 и 1 оно делится также на 2 и 4.

Если перемножить два простых числа, то полученное произведение больше никак нельзя получить другим способом (кроме умножения этого же числа на единицу). Поясним на примере.

Возьмём два простых числа 5 и 7 и перемножим их — получится 35. Больше число 35 получить никак не получится, кроме как умножить 35 на 1. Это значит, что если произведение можно разложить на два простых множителя, то других вариантов разложения (кроме числа и единицы) у него не будет. Это нам пригодится при решении задач — и если число можно разложить на 2 простых, то и их сумму тоже легко сразу посчитать.

Ещё пример:

54 = 2 × 27

54 = 3 × 18

54 = 6 × 9, а это значит, что число 54 нельзя получить перемножением двух простых чисел и нельзя сразу сказать, чему однозначно равна сумма множителей.

И ещё:

21 = 3 × 7

Оба числа простые, поэтому произведение 21 можно получить только из них, а значит, легко посчитать сумму — она будет равна 3 + 7 = 10.

Теперь переведём их диалог на язык математики и логики и обозначим числа как n и m:

Первый: Я понял, что одно из чисел точно не простое, потому что иначе я сразу бы разложил число на произведение двух простых и легко получил сумму. А раз так, то это одно из чисел m или n можно получить перемножением двух других чисел. Поэтому общее произведение состоит не менее чем из трёх множителей, причём как минимум один из них отличается от остальных — поэтому получается несколько вариантов возможных сумм, и я не знаю, какая из них правильная (пометим это как Правило 1).

Второй: Сумму, которая у меня есть, нельзя получить из двух простых чисел, поэтому и твоё произведение тоже нельзя разложить на два простых множителя. Это значит, что у меня нечётная сумма, потому что, по гипотезе Гольдбаха, в нашем случае можно получить любое чётное число, сложив два простых. А раз это не два простых числа, значит, и сумма будет нечётная. А ещё эта сумма точно не равна сумме двух и простого числа, потому что два — тоже простое, ха! Поэтому есть несколько вариантов суммы m и n, которые подходят под твои условия, но я не могу пока определить, какие именно (пометим это как Правило 2).

Первый: Из всех множителей моего произведения я могу составить только один вариант пары, сумма которой подойдёт под твоё ограничение — не будет разбиваться на сумму двух простых или сумму чисел одного множителя (Правило 3).

Второй: Ах вот как! Из всех вариантов пар, на которые можно разбить сумму и подходящих под твои условия, есть только одна, которая позволила бы тебе определить это (Правило 4). Теперь и мне понятно, что это за числа!

Теперь подберём варианты суммы, которая была у второго. Ограничения такие:

  • нечётная;
  • не равна сумме двойки и простого числа.

1 — не подходит, потому что оба числа больше единицы.

2, 4, 6, 8… — нет, потому что чётные.

3 — нет, потому что это сумма двойки и простого числа.

5 — нет, по той же причине (2 + 3).

7 — тоже нет (2 + 5).

9 — тоже нет (2 + 7, а 7 — простое число).

11 — подходит.

13 — нет, потому что 13 = 2 + 11 (11 — простое число).

15 — нет, потому что 15 = 2 + 13 (13 — тоже простое число).

17 — подходит.

19 — нет, потому что 19 = 2 + 17 (17 — простое число).

Способ подбора суммы понятен, дальше можно продолжать по тому же алгоритму. Мы же выберем те, которые нам уже подошли, и на их примере покажем, что нужно делать дальше, чтобы получить правильный ответ. Наши числа, которые нам подходят уже сейчас: 11 и 17. Начнём с 11.

Сумма = 11.

Найдём все слагаемые, которые могут давать эту сумму:

2 + 9

3 + 8

4 + 7

5 + 6

Для каждого из них запишем произведение и проверим, выполняется ли Правило 3, которое сказал первый программист.

Смотрим на произведение 2 × 9 = 18 и как ещё его можно получить.

18 = 2 × 9 → Да (Правило 3 выполняется).

18 = 3 × 6 → Нет (Правило 3 не работает, потому что 3 + 6 = 9, а 9 можно получить из простых чисел 2 и 7).

Смотрим на произведение 3 × 8 = 24.

24 = 2 × 12 → Нет (чётная сумма, Правило 2 не работает).

24 = 3 × 8 → Да (выполняется Правило 3).

24 = 6 × 4 → Нет (чётная сумма).

Смотрим на произведение 4 × 7 = 28.

28 = 2 × 14 → Нет (чётная сумма).

28 = 4 × 7 → Да (выполняется Правило 3).

Смотрим на произведение 5 × 6 = 30.

30 = 2 × 15 → Да.

30 = 3 × 10 → Нет (Правило 3 не работает, потому что 3 + 10 = 13, а 13 можно получить суммой простых чисел 2 и 11).

30 = 5 × 6 → Да.

Тут мы вообще не можем выбрать одну пару, потому что Правило 3 выполняется 2 раза, а значит, этот вариант отбрасываем.

Получается, что для суммы 11 могут быть три варианта произведений, для которых выполняется Правило 3: 2 и 9, 3 и 8, 4 и 7. Но тогда Правило 4 не выполняется, потому что нужно, чтобы для одной суммы была только одна пара, которая подходит под правило 3. Продолжаем искать.

Сумма = 17.

Найдём все слагаемые, которые могут давать эту сумму:

2 + 15

3 + 14

4 + 13

5 + 12

6 + 11

7 + 10

8 + 9

Для каждого из них запишем произведение и проверим, выполняется ли Правило 3, которое сказал первый программист.

Смотрим на произведение 2 × 15 = 30 и как ещё его можно получить.

30 = 2 × 15 → Да.

30 = 3 × 10 → Нет (Правило 3 не работает, потому что 3 + 10 = 13, а 13 можно получить суммой простых чисел 2 и 11).

30 = 5 × 6 → Да.

Тут мы вообще не можем выбрать одну пару, потому что Правило 3 выполняется 2 раза, а значит, этот вариант отбрасываем.

 

Смотрим на произведение 3 × 14 = 42 и как ещё его можно получить:

42 = 2 × 21 → Да.

42 = 3 × 14 → Да.

42 = 6 × 7 → Нет.

Два раза выполняется Правило 3 — отбрасываем пару.

Смотрим на произведение 4 × 13 = 52 и как ещё его можно получить.

52 = 2 × 26 → Нет.

52 = 4 × 13 → Да.

Смотрим на произведение 5 × 12 = 60 и как ещё его можно получить.

60 = 2 × 30 → Нет.

60 = 3 × 20 → Да.

60 = 5 × 12 → Да.

60 = 6 × 10 → Нет.

Два раза выполняется Правило 3 — отбрасываем пару.

Смотрим на произведение 6 × 11 = 66 и как ещё его можно получить.

66 = 2 × 33 → Да.

66 = 3 × 22 → Нет.

66 = 6 × 11 → Да.

Два раза выполняется Правило 3 — отбрасываем пару.

Смотрим на произведение 7 × 10 = 70 и как ещё его можно получить.

70 = 2 × 35 → Да.

70 = 5 × 14 → Нет.

70 = 7 × 10 → Да.

Два раза выполняется Правило 3 — отбрасываем пару.

Смотрим на произведение 8 × 9 = 72 и как ещё его можно получить.

72 = 2 × 36 → Нет.

72 = 3 × 24 → Да.

72 = 4 × 18 → Нет.

72 = 6 × 12 → Нет.

72 = 8 × 9 → Да.

Два раза выполняется Правило 3 — отбрасываем пару.

Получается, что для суммы 17 может быть только один вариант произведения, для которого выполняется Правило 3: это 4 и 13. А значит, что Правило 4 тоже выполняется и мы нашли нужные числа!

Если вы дочитали досюда и всё поняли — снимаем шляпу. Вы не из тех, кого могут испугать вычисления и логический подход!

ГДЗ по Математике 5‐6 класс задачи на смекалку Шарыгин

Автор: Шарыгин И.Ф..

Одним из дополнительных методических пособий, используемых в школах, гимназиях и лицеях в дополнение к учебнику, является материал, ответы к которому вы можете найти в режиме онлайн ГДЗ по математике 5‐6 класс задачи на смекалку Шарыгин. Книга была издана в 2018 году издательством «Просвещение». Безусловно, очень интересное и полезное пособие, которое включает:

  • задачи на составление разного рода выражений;
  • обнаружение чисел;
  • разрезание фигур на равные части;
  • различные ребусы, головоломки и задачи.

Первые годы обучения в старшей школе являются самыми сложными, поскольку до сих пор у школьников происходит добавление новых дисциплин и учебных предметов. Также большое количество новых терминов появится и в уже изучаемых предметах, закрепление ранее усвоенной информации является тоже очень важным, поскольку именно правильно сформированная база знаний обеспечит успешное изучение нового материала.

Учеба с ГДЗ по математике к задачам на смекалку для 5‐6 класса Шарыгина

Школьная программа пятиклассников полна новыми понятиями и терминами, благодаря этому большинство учеников сталкивается с первыми трудностями. Подростки узнают о дробных числах, разберутся, в чем отличие правильных и неправильных дробей, также ребята познакомятся с десятичными дробями, поймут, как осуществляется их сложение и вычитание, деление и умножение. Основные темы пятого года обучения:

  • понятие натуральных чисел;
  • сложение и вычитание натуральных чисел;
  • умножение и деление натуральных чисел;
  • углубление знаний о геометрических фигурах, более подробное изучение мер площади и объема.

Также учеников познакомят с основами комбинаторики и различными инструментами, которые способствуют вычислениям и измерениям. Помимо основных учебных материалов многие школы используют материалы дополнительные, которые не только углубляют программу изучения, но и существенно расширяют ее границы.

Важно отметить, что в конце главной книги даны решения лишь к некоторым из номеров, для того, чтобы получить ответ на другие, можно лишь воспользоваться советом по решению и воспользоваться смекалкой. Решебник по математике для задач на смекалку за 5‐6 класс (автор: Шарыгин И. Ф.) включает в себя полное решение, ученики получат не только ответ, но и ход вычислений в задании. Что позволит понять, почему в решении определенного задания возникала сложность.

20 сложных, но забавных вопросов по математике для начальной школы

Если вы не выросли инженером, банкиром или бухгалтером, велика вероятность, что математика в начальной и средней школе была проклятием вашего существования. Вы будете без устали готовиться неделями к этим глупым стандартизированным тестам — и, тем не менее, придя к экзамену, вы так или иначе не поймете, о чем просили какие-либо уравнения или сложные математические задачи. Поверьте, мы это понимаем.

Хотя логика может привести вас к мысли, что ваши математические навыки естественным образом улучшились с возрастом, печальная реальность такова, что, если вы не решаете задачи алгебры и геометрии на ежедневной основе, скорее всего обратное. .

Не верите нам? Затем проверьте свою мудрость с помощью этих сложных математических вопросов, взятых прямо из школьных тестов и домашних заданий, и убедитесь в этом сами.

1. Вопрос: Какое количество парковочных мест занято автомобилем?

Эта сложная математическая задача стала вирусной несколько лет назад после того, как появилась на вступительном экзамене в Гонконге… для шестилетних детей. Якобы у студентов было всего 20 секунд, чтобы решить задачу!

Ответ: 87.

Вы не поверите, но этот «математический» вопрос на самом деле не требует никаких математических вычислений. Если вы перевернете изображение вверх ногами, вы увидите, что вы имеете дело с простой числовой последовательностью.

2. Вопрос: Замените вопросительный знак в указанной выше проблеме на соответствующий номер.

Эту проблему не должно быть слишком трудно решить, если вы много играете в судоку.

Ответ: 6.

Сумма всех чисел в каждой строке и столбце дает 15! (Кроме того, 6 — единственное число, не представленное из чисел от 1 до 9.)

3. Вопрос: Найдите эквивалентное число.

Эта проблема возникла прямо из стандартного теста, проведенного в Нью-Йорке в 2014 году.

Ответ: 9.

Shutterstock

Простите, если вы точно не помните, как работают экспоненты. Чтобы решить эту проблему, вам просто нужно вычесть экспоненты (4-2) и решить для 3 2 , которое расширяется до 3 x 3 и равно 9.

4. Вопрос: Сколько маленьких собак зарегистрировано для участия в выставке?

Изображение предоставлено Imgur / zakiamon

Этот вопрос взят непосредственно из домашнего задания второклассника по математике.Ой.

Ответ: 42,5 собаки.

Чтобы определить, сколько маленьких собак соревнуются, вы должны вычесть 36 из 49 и затем разделить полученный ответ 13 на 2, чтобы получить 6.5 собак, или количество соревнующихся крупных собак. Но вы еще не закончили! Затем вам нужно добавить 6,5 к 36, чтобы получить количество соревнующихся маленьких собак, которое составляет 42,5. Конечно, на самом деле половина собаки не может участвовать в выставке собак, но ради этой математической задачи давайте предположим, что это так.

5. Вопрос: Найдите площадь красного треугольника.

Изображение с YouTube

Этот вопрос использовался в Китае для выявления одаренных пятиклассников. Предположительно, некоторые из умных студентов смогли решить эту проблему менее чем за одну минуту.

Ответ: 9.

Чтобы решить эту проблему, вам необходимо понять, как работает площадь параллелограмма. Если вы уже знаете, как связаны площадь параллелограмма и площадь треугольника, тогда добавление 79 и 10 и последующее вычитание 72 и 8, чтобы получить 9, должно иметь смысл, но если вы все еще не уверены, то посмотрите этот YouTube видео для более подробного объяснения.

6. Вопрос: Какой высоты стол?

Изображение с YouTube

YouTube MindYourDecisions адаптировал этот ошеломляющий математический вопрос из аналогичного, найденного в домашнем задании ученика начальной школы в Китае.

Ответ: 150 см.

Изображение с YouTube

Поскольку одно измерение включает в себя рост кошки и вычитает рост черепахи, а другое дает обратное, вы можете просто действовать так, как будто двух животных нет. Поэтому все, что вам нужно сделать, это сложить два измерения — 170 см и 130 см — и разделить их на 2, чтобы получить высоту стола 150 см.

7. Вопрос: Если стоимость биты и бейсбольного мяча вместе составляет 1,10 доллара, а бита стоит на 1 доллар больше, чем мяч, сколько стоит мяч?

Shutterstock

С математической точки зрения эта задача очень похожа на одну из других задач в этом списке.

Ответ: 0,05 доллара.

Вернитесь к задаче о собаках на выставке и используйте ту же логику, чтобы решить эту проблему. Все, что вам нужно сделать, это вычесть 1 доллар из 1,10 доллара и затем разделить полученный ответ, 0,10 доллара на 2, чтобы получить окончательный ответ — 0 долларов.05.

8. Вопрос: Когда у Шерил день рождения?

Изображение через Facebook / Kenneth Kong

Если у вас возникли проблемы с прочтением, см. Здесь:

«Альберт и Бернард только что подружились с Шерил, и они хотят знать, когда у нее день рождения. Черил дает им список из 10 возможных дат.

15 мая 16 мая 19 мая

17 июня 18 июня

14 июля 16 июля

14 августа 15 августа 17 августа

Затем Шерил сообщает Альберту и Бернарду отдельно месяц и день своего дня рождения соответственно.

Альберт: Я не знаю, когда у Шерил день рождения, но я знаю, что Бернард тоже не знает.

Бернард: Сначала я не знал, когда у Шерил день рождения, но теперь знаю.

Альберт: Тогда я также знаю, когда у Шерил день рождения.

Так когда же день рождения Шерил? »

Непонятно, почему Шерил не могла просто сказать Альберту и Бернарду месяц и день своего рождения, но это не имеет отношения к решению этой проблемы.

Ответ: 16 июля.

Не знаете, как найти ответ на этот вопрос? Не волнуйтесь, таково было большинство людей в мире, когда этот вопрос, взятый из олимпиады по математике в Сингапуре и азиатских школах, стал вирусным несколько лет назад.К счастью, New York Times шаг за шагом объясняет, как добраться до 16 июля, и вы можете прочитать их подробный вывод здесь.

9. Вопрос: Найдите пропущенную букву.

Изображение через Facebook / Семья Холдернесса

Это взято из домашнего задания первоклассника .

Ответ: Отсутствует буква J.

.

Когда вы складываете значения, указанные для S, B и G, сумма получается 40, и если недостающая буква J (которая имеет значение 14) делает сумму другой диагонали такой же.

10. Вопрос: Решите уравнение.

Изображение с YouTube

Эта проблема может показаться простой, но удивительное количество взрослых не могут ее решить правильно.

Ответ: 1.

Начните с решения части уравнения с делением. Для этого, если вы забыли, вам нужно перевернуть дробь и переключиться с деления на умножение, получив 3 x 3 = 9. Теперь у вас есть 9 — 9 + 1, и оттуда вы можете просто работать слева вправо и получите окончательный ответ: 1.

11. Вопрос: Где должна быть проведена линия, чтобы уравнение ниже было точным?

5 + 5 + 5 + 5 = 555.

Ответ: На знаке «+» должна быть проведена линия.

Когда вы рисуете наклонную линию в верхнем левом квадранте знака «+», она становится числом 4, и уравнение, таким образом, принимает вид 5 + 545 + 5 = 555.

12. Вопрос: Решите незаконченное уравнение.

Попытайтесь выяснить, что общего у всех уравнений.

Ответ: 4 = 256.

Формула, используемая в каждом уравнении: 4 x = Y. Итак, 4 1 = 4, 4 2 = 16, 4 3 = 64 и 4 4 = 256,

13. Вопрос: Сколько треугольников на изображении выше?

Когда Best Life впервые написал об этом обманчивом вопросе, нам пришлось попросить математика объяснить ответ!

Ответ: 18.

Некоторых людей ставят в тупик треугольники, прячущиеся внутри треугольников, а другие забывают включить гигантский треугольник, в котором находятся все остальные. В любом случае, очень немногие люди — даже учителя математики — смогли найти правильный ответ на эту проблему. А чтобы узнать о других вопросах, которые будут проверять ваше прежнее образование, ознакомьтесь с этими 30 вопросами, которые вам понадобятся для успешной сдачи 6-го класса по географии.

14. Вопрос: сложите 8,563 и 4,8292.

Сложить два десятичных знака проще, чем кажется.

Ответ: 13.3922.

Пусть вас не сбивает с толку тот факт, что у 8.563 меньше чисел, чем у 4.8292. Все, что вам нужно сделать, это добавить 0 в конец 8.563, а затем добавить, как обычно.

15. Вопрос: На озере есть участок с кувшинками. Каждый день нашивка увеличивается в размерах вдвое…

Shutterstock

… Если заплатке требуется 48 дней, чтобы покрыть все озеро, сколько времени потребуется, чтобы заплатка покрыла половину озера?

Ответ: 47 дн.

Большинство людей автоматически предполагают, что половина озера будет покрыта за половину времени, но это предположение неверно.Поскольку участок площадок удваивается в размере каждый день, озеро будет наполовину покрыто всего за день до того, как оно покроется полностью.

16. Вопрос: Сколько футов в миле?

Эта задача уровня начальной школы представляет собой немного меньше решения задач и немного больше запоминания.

Ответ: 5280.

Это был один из вопросов, представленных в популярном шоу «» Вы умнее пятиклассника?

17. Вопрос: Какое значение «x» делает приведенное ниже уравнение истинным?

Shutterstock

-15 + (-5x) = 0

Ответ: -3.

Вас простят за то, что вы думаете, что ответ был 3. Однако, поскольку число рядом с x отрицательно, нам нужно, чтобы x также был отрицательным, чтобы получить 0. Следовательно, x должен быть -3.

18. Вопрос: Сколько 1,92 делится на 3?

Возможно, вам придется попросить помощи у ваших детей.

Ответ: 0,64.

Чтобы решить эту, казалось бы, простую проблему, вам нужно удалить десятичную дробь из 1,92 и действовать так, как будто ее там нет. После того, как вы разделите 192 на 3, чтобы получить 64, вы можете вернуть десятичный знак на место и получить окончательный ответ 0.64.

19. Вопрос: Решите математическое уравнение выше.

Изображение с YouTube

Не забывайте о PEMDAS!

Ответ: 9.

Используя PEMDAS (аббревиатура, указывающая порядок, в котором вы ее решаете: «скобки, показатели, умножение, деление, сложение, вычитание»), вы сначала решаете сложение внутри круглых скобок (1 + 2 = 3) и оттуда закончите уравнение, как оно написано слева направо.

20. Вопрос: Сколько там зомби?

Чтобы найти ответ на этот последний вопрос, потребуется использовать дроби.

Ответ: 34.

Поскольку мы знаем, что на каждые три человека приходится два зомби и что 2 + 3 = 5, мы можем разделить 85 на 5, чтобы вычислить, что всего существует 17 групп людей и зомби. Затем мы можем умножить 17 на 2 и 3 и узнать, что существует 34 зомби и 51 человек соответственно. Не так уж и плохо, правда?

Чтобы узнать больше удивительных секретов о том, как прожить свою лучшую жизнь, нажмите здесь , чтобы подписаться на нас в Instagram!

Бесплатные задания по математике для 6-го класса

Вы здесь: Главная → Задания → 6 класс

Это исчерпывающая коллекция бесплатных распечатываемых рабочих листов по математике для шестого класса, организованных по таким темам, как умножение, деление, показатели, разряд, алгебраическое мышление, десятичные дроби, единицы измерения, соотношение, процент, разложение на простые множители, GCF, LCM, дроби, целые числа и геометрия.Они генерируются случайным образом, их можно распечатать в вашем браузере и включать в себя ключ ответа. Рабочие листы подходят для любой математической программы шестого класса, но особенно хорошо подходят для математической программы IXL для шестого класса.

Рабочие листы генерируются случайным образом каждый раз, когда вы нажимаете на ссылки ниже. Вы также можете получить новый, другой, просто обновив страницу в своем браузере (нажмите F5).

Вы можете распечатать их прямо из окна браузера, но сначала проверьте, как это выглядит в «Предварительном просмотре».Если рабочий лист не умещается на странице, отрегулируйте поля, верхний и нижний колонтитулы в настройках страницы вашего браузера. Другой вариант — настроить «масштаб» на 95% или 90% в предварительном просмотре печати. В некоторых браузерах и принтерах есть опция «Печатать по размеру», которая автоматически масштабирует рабочий лист по размеру области печати.

Все рабочие листы содержат ключ ответа на 2-й странице файла.

В шестом классе ученики начнут изучать алгебру для начинающих (порядок операций, выражения и уравнения).Они узнают о соотношениях и процентах и ​​начинают использовать целые числа. Студенты также изучают деление на множители, факторизацию, арифметику дробей и десятичную арифметику. В геометрии основное внимание уделяется площади треугольников и многоугольников и объему прямоугольных призм. Другие темы включают округление, экспоненты, GCF, LCM и единицы измерения. Обратите внимание, что эти бесплатные рабочие листы не охватывают все темы 6-го класса; в первую очередь, они не включают решение проблем.

Умножение и деление и некоторые обзоры

Длинное умножение

Длинное деление

  • 1-значный делитель, 5-значное делимое, без остатка
  • 1-значный делитель, 5-значное делимое, с остатком
  • 1-значный делитель, 6-значное делимое, без остатка
  • 1-значный делитель, 6-значное делимое, с остатком
  • 1-значный делитель, 7-значное делимое, без остатка
  • 1-значный делитель, 7-значное делимое, с остатком
  • 2-значный делитель, 5-значное делимое, без остатка
  • 2-значный делитель, 5-значное делимое, с остатком
  • 2-значный делитель, 6-значное делимое, без остатка
  • 2-значный делитель, 6-значное делимое, с остатком
  • 2-значный делитель, 7-значное делимое, без остатка
  • 2-значный делитель, 7-значное делимое, с остатком
  • 3-значный делитель, 6-значное делимое, без остатка
  • 3-значный делитель, 6-значное делимое, с остатком
  • 3-значный делитель, 7-значное делимое, без остатка
  • 3-значный делитель, 7-значное делимое, с остатком
  • Умножение десятичных знаков, запись чисел друг под другом (0–2 десятичные цифры)
  • Разделите целое число или десятичную дробь на целое число, к делимому нужно добавить нули
  • Преобразование дроби в десятичную дробь с помощью длинного деления с округлением ответов до трех знаков после запятой

Преобразование единиц измерения с помощью деления в столбик и умножения

Математика для начальных классов Эдвард Заккаро

Хорошая книга по решению проблем с очень разнообразными текстовыми задачами и стратегиями решения проблем.Включает главы по следующим темам: последовательности, решение проблем, деньги, проценты, алгебраическое мышление, отрицательные числа, логика, отношения, вероятность, измерения, дроби, деление. Вопросы в каждой главе разбиты на четыре уровня: легкий, несколько сложный, сложный и очень сложный.

Экспоненты

Разрядное значение / округление

Алгебра

Порядок работы

  • Три операции, использует ÷ для деления, без показателей
  • Четыре операции, использует ÷ для деления, без показателей
  • Две или три операции, для деления используется дробная линия, без показателей
  • Две или три операции, для деления используется дробная линия, включая показатели степени
  • Две, три или четыре операции, используется дробная линия, включая показатели степени

Выражения

Уравнения

Ключ к учебным пособиям по алгебре

Key to Algebra предлагает уникальный проверенный способ познакомить студентов с алгеброй.Новые концепции объясняются простым языком, а примеры легко следовать. Задачи со словами связывают алгебру с знакомыми ситуациями, помогая учащимся понять абстрактные концепции. Учащиеся развивают понимание, интуитивно решая уравнения и неравенства, прежде чем будут представлены формальные решения. Студенты начинают изучение алгебры с книг 1–4, используя только целые числа. Книги 5-7 вводят рациональные числа и выражения. Книги 8-10 охватывают реальную систему счисления.

=> Узнать больше

Дроби vs.Десятичные

  • Десятичные дроби или смешанные числа (десятые / сотые / тысячные)
  • Десятичные дроби и смешанные числа (с точностью до миллионной)
  • Смешанные числа с десятичными знаками (знаменатели 10, 100 и 1000)
  • Правильные и неправильные дроби с десятичными знаками (знаменатели 10, 100 или 1000)
  • Правильные дроби до десятичных (знаменатели в степени от десяти до 1000000)
  • Смешанные числа с десятичными знаками (знаменатели со степенями от десяти до 1000000)
  • Дроби или смешанные числа с десятичными знаками (простые, различные знаменатели)
  • Дробь в десятичную дробь — нужно деление в столбик
  • Дроби в десятичные дроби — смешанная практика

Сложение и вычитание десятичных чисел

Ключ к книгам с десятичными знаками

Это серия учебных пособий компании Key Curriculum Press, которая начинается с основных понятий и операций с десятичными знаками.Затем книги охватывают реальное использование десятичных дробей в ценообразовании, спорте, метриках, калькуляторах и науке.

В комплекте книги 1-4.

=> Узнать больше

Десятичное умножение

Умножение умственного развития

Умножить по столбцам

Десятичное деление

Психологическое отделение

  • Простое десятичное деление (делимое состоит из 1-2 десятичных цифр, делитель целого числа)
  • То же, что и выше, но без дивиденда или делителя
  • Разделите десятичные дроби на десятичные (подумайте, сколько раз делитель вписывается в частное.)
  • Смешанные задачи умножения и деления 1 (1 десятичная цифра)
  • Разделите целые и десятичные числа на 10, 100 или 1000
  • То же, что и выше, без дивиденда или делителя
  • Умножение или деление десятичных и целых чисел на 10, 100 и 1000
  • Разделите целые и десятичные числа на 10, 100, 1000 или 10 000
  • Разделите целые и десятичные числа на 10, 100, 1000 или 10 000 — делимое или делитель отсутствует

Длинное деление

Единицы измерения

Обычная система

Преобразование единиц измерения с помощью деления в столбик и умножения (бумага и карандаш) или умственной математики

Преобразование с помощью калькулятора с десятичными знаками

Метрическая система

  • Преобразование между мм, см и м — с использованием десятичных знаков
  • Преобразование между мм, см, м и км — с использованием десятичных знаков
  • Преобразование между мл и л и г и кг — с использованием десятичных знаков
  • Все упомянутые выше метрические единицы — смешанная практика — с использованием десятичных знаков
  • Метрическая система: перевод единиц длины (мм, см, дм, м, плотина, гм, км)
  • Метрическая система: перевод единиц веса (мг, cg, dg, g, dag, hg, kg)
  • Метрическая система: преобразование единиц объема (мл, кл, дл, л, дал, гл, кл)
  • Метрическая система: преобразование единиц длины, веса и объема

Коэффициент

Процент

Факторизация на простые множители, GCF и LCM

Сложение и вычитание дробей

Умножение на дроби

Во всех задачах умножения и деления дробей это помогает упростить, прежде чем умножать.

Фракционное подразделение

Преобразование дробей в смешанные числа и vv

Упрощенная дробь или эквивалентная дробь

Дроби и десятичные числа

Целые

Сетка координат

Сложение и вычитание

Сложение и вычитание целых чисел выходят за рамки Общих основных стандартов для 6 класса, но некоторые учебные программы или стандарты могут включать их в 6-м классе.

Умножение и деление

Умножение и деление целых чисел выходят за рамки Общих основных стандартов для 6-го класса, но ссылки на рабочие листы включены сюда для полноты, так как некоторые учебные программы или стандарты могут включать их в 6-м классе.

Геометрия

Область — эти рабочие листы выполняются в координатной сетке.

Объем и площадь поверхности

Поскольку эти листы ниже содержат изображения различных размеров, сначала проверьте
как выглядит рабочий лист в предварительном просмотре перед печатью. Если это не так
подходит, вы можете либо распечатать его в масштабе (например, 90%), либо сделать еще один,
обновляйте страницу рабочего листа (F5), пока не получите подходящую.

Дополнительные темы

Пропорции

Круг


Если вы хотите иметь больший контроль над такими параметрами, как количество задач, размер шрифта, интервал между проблемами или диапазон чисел, просто
щелкните по этим ссылкам, чтобы самостоятельно использовать генераторы рабочих листов:

Сложные задачи по математике для 6-го класса — 6 класс

6tth 6 — вот где мы действительно начинаем заниматься алгеброй.Не только решение для x, но и использование переменных в форме выражения. Основной навык предалгебры — понять, как использовать переменные для управления уравнениями и выражениями.

Задачи по математике для 6-х классов — Задания по математике для 6-х классов

Это новая страница хаба, которая в настоящее время находится в стадии разработки, так что скоро появятся новые ресурсы для 6-го класса! Здесь вы найдете бесплатные распечатанные рабочие листы по математике 6-го класса. Оценка соответствует стандартам для 6-го класса.Приходите и взгляните на нашу страницу добавления вычитания дробей или нашу страницу математических игр для 6-го класса. Нужна помощь. Проблемы алгебры? У нас также есть некоторые базовые рабочие листы по алгебре.

сложных математических задач для шестиклассников — задания по математике для шестого класса, игры, задачи и многое другое!

Это сборник бесплатных видеороликов по математике для 6-х классов, в которых показаны упражнения для оценок по каждой теме. Они соответствуют учебной программе 6-го класса по математике Mammoth, но также будут работать независимо от того, какой учебный план вы следуете, другими словами, 6-й не зависит от того, что у вас есть учебная программа по Math Mammoth.Выражения — что такое выражения. Как написать технологический документ и как их оценивать. Написание и упрощение выражений 1: длина и периметр.

Задачи по математике для 6-х классов — Задания по математике для 6-го класса PDF | Задания по математике для шестого класса с ответами

Если так, то дальше не ищите. У нас есть коллекция бесплатных заданий и рабочих листов, которые помогут вашим ученикам подготовиться к 6 классу по математике и задачам. Вы можете скачать рабочие листы и распечатать их столько, сколько вам нужно.У вас есть разрешение на распространение печатных материалов по математике среди выпускников, учителей, репетиторов и друзей. Проблема У вас нет разрешения на отправку этих листов кому-либо каким-либо образом по электронной почте, текстовым сообщениям или другим способом.

Представьте себе открытые математические задачи, в которых учащиеся задают вопросы, а также отвечают на них. Творческие математические подсказки побуждают учащихся строить и проверять свои собственные идеи. Черепаха и Заячья гонка.

Рабочие листы с заданиями по математике для 6-х классов — Бесплатные видеоуроки по математике для 6-х классов

Введите свой математический номер или адрес проблемы ниже, и мы отправим вам ссылку для загрузки бесплатного приложения Kindle.Затем вы с шестого числа начинаете читать Kindle Grade на своем смартфоне, планшете или компьютере — устройство Kindle не требуется. Чтобы получить бесплатное приложение, введите номер своего мобильного телефона. Развивайте навыки решения проблем с помощью этой уникальной серии, в которой используются художественные и научно-популярные тексты для представления математических задач и ситуаций. Студенты должны прочитать отрывки, а затем использовать критическое мышление для выполнения каждого задания.

Задачи по математике для 6-го класса — Интерактивные математические навыки для 6-го класса — Задачи со словами

31 декабря, — Area Invasion — это настольная игра, в которой ученики 6-го класса тренируются находить прямоугольники, параллелограммы, треугольники, трапеции и математику.Этот ресурс включает печатную и цифровую версии. Студенты должны найти области различной формы, чтобы помочь им перемещать Proboems по игровому полю. Выбор «Этот набор математических средних значений для 6-го класса, Темы для эссе аргументов для средних, режимов и диапазонов рабочих листов колледжа помогает учащимся практиковать эти концепции оценок. Пакет для обзора Gradw легко подготовить и включает в себя печатные и цифровые слайды Google.

сложных математических задач для 6-х классов — Лучшие бесплатные ресурсы по математике для 6-х классов: полный список! — Mashup Math

Узнайте больше о файлах cookie.Эти наши рабочие листы для печати по математике для 6-го класса охватывают всю основную математику для 6-го класса, некоторые из которых включают; Целые числа, умножение, деление, экспоненты. Проблемы с квадратными корнями, теория чисел, десятичные дроби, сложение и вычитание десятичных дробей, умножение и деление десятичных дробей, дроби и смешанные числа, сложение и вычитание дробей, умножение дробей. Занимая 6-е место, мы можем похвастаться забавными рисунками, хорошо структурированными упражнениями и интересными задачами из реальной жизни.Этот класс очень вдохновит учеников 6 класса по математике на непреодолимое желание решить задачи по математике.

Математика для начальных классов, Эдвард Заккаро. Хорошая книга по решению проблем с очень разнообразными текстовыми задачами и стратегиями решения проблем. Лист оценки выражений · Факторинг / Распределение · Рабочий лист области. 6-й класс · Краткое содержание курса · Коэффициенты, ставки и проценты · Арифметические операции · Отрицательные числа · Свойства чисел · Переменные и выражения · Уравнения и уравнения и неравенства · Коэффициенты, ставки и проценты · Арифметические операции.

Эти ссылки побуждают учащихся развивать логическое мышление и использовать шаблоны для решения этих головоломок и головоломок. Эти ссылки ведут на общие сайты по подготовке к тестам, которые предоставляют вопросы в формате с несколькими вариантами ответов, на которые учащиеся 6-го класса отвечают в Интернете. Домашняя математика Задачи по математике. Оценки Эти задачи предназначены для учащихся, учащихся 6-го класса «Дробление шаблонов блоков» для оценки фигуры с помощью блоков шаблонов для создания дизайна, отвечающего определенным требованиям. Spinner Game представляет учащимся Таблицу оценок вращений и ученикам-математикам, чтобы создать график Grdae результатов и нарисовать счетчик, который даст эти результаты.

Просмотреть проблемуx. Проблемы с десятичным словом. Работаю 8 часов в день. Какое десятичное число проиллюстрировано практикой, Чтение и запись десятичных знаков в словесной практике, Как разместить десятичные дроби на практике числовой линии.

6 страница математики. Практикуйте математику онлайн с неограниченным количеством вопросов по математике более чем за 6 лет Умножение чисел, заканчивающихся нулями: проблемы со словами. B Рабочие листы по математике для 6-го класса: числовое значение и научная запись, умножение и деление, дроби и десятичные дроби, факторинг, пропорции, показатели степени, целые числа, проценты.

Задачи и ответы по математике для 6-го класса — 30 вопросов, которые вам понадобятся, чтобы сдать 6-й класс по математике | Лучшая жизнь

Сообщение: Энтони Персико. Вы ищете лучшие бесплатные ресурсы по математике для 6-го класса, планы уроков, идеи 6hh и рабочие листы — все в одном Задачи В следующем списке представлены некоторые из лучших, популярных и математических ресурсов, таких как бесплатные рабочие листы по математике для шестого класса и ключи для 6-го класса. Учителя 6-го класса и родители в классе, дистанционном обучении и домашнем обучении.Бесплатные рабочие листы с задачами по математике для 6-го класса.

Математические задачи со словами Рабочие листы Калькуляторы. Задачи — 6-й класс 11лет — математические задачи Число найденных 6-го: 70 Скорость Грейи За сколько минут часовая стрелка пройдет математический угол и прямой угол? Вентилятор Вентилятор имеет номинальную скорость об / мин. Рассчитайте на время одного периода вентилятора.

Задачи и ответы по математике для 6 класса — Скорость — 6 класс (11 лет) — Задачи по математике

Студенты-математики познакомились с концепцией соотношения. Оценка использует рассуждения об умножении и делении для решения задач на соотношение.Учащиеся развивают свое понимание соотношений к подсказкам для эссе «Причина и следствие», развивают 6-ю оценку, единичные задачи и проценты. Работа с соотношениями и пропорциями предоставляет контекст для создания выражений и уравнений.

Рабочие листы и печатные формы по математике для второго класса. Просмотрите перегруппировку задач вычитания. Дети могут оценить свои навыки перегруппировки на этой таблице вычитания. Этот рабочий лист по математике дает много времени для практики для детей 6-го, которые нуждаются в дополнительном подталкивании.

Задачи по математике для 6-х классов — Общие основные стандарты 6-го класса

Вы, вероятно, читаете это прямо сейчас в стрессе, разочаровании, истощении и т. Д. Я чувствую вас — и как человек, который занимался проблемами 7 лет, я абсолютно понимаю, где вы сейчас находитесь. Шестой, генеральный директор ведущей компании по обучению 6-х и выпускников в стране, я считаю своим долгом помочь всем ученикам 6-х классов улучшить свои математические навыки, независимо от того, пользуются они нашими услугами или нет.Поэтому команда математиков и я потратили десятки часов исследований, чтобы составить регулярно обновляемый пост обо всем, что вам нужно знать о математике для 6-го класса, из наиболее достоверных проблем, доступных в настоящее время в Интернете, в практическом и увлекательном формате.

Нет проблем! Пакет оценок. Teachers Pay Teachers — это онлайн-торговая площадка, где учителя покупают и продают оригинальные учебные материалы.

Пропорции велосипеда грязи. Дроби единицы — это дроби со знаменателем 1.Дети очень хорошо относятся к играм. Войдите в систему с другой учетной записью 6rh account.

Задачи на дробное слово 6 класс

Если вы — Grsde, чтобы произвести впечатление на своего ребенка, и математику с его домашним заданием, подумайте о распространенных математических задачах. Коэффициенты — это одна из областей, где оценка встречается с реальным миром, и иногда дети борются с проблемами именно в этом классе 6-го класса. Если всего 65 учеников, сколько мальчиков будут шестыми в классе? Чтобы решить, вам нужно найти соотношение Задач к общему количеству студентов.Теперь вам нужно создать еще одну фракцию в 6 трлн, используя имеющуюся у вас информацию.

Получите крупные скидки на Instant Pot, матрас из пены с эффектом памяти и многое другое в президентском классе. Хотите помочь своему шестикласснику освоить математику? Вот некоторые из навыков, которые ваш шестой ученик будет изучать в классе математики. В стаде из 6-го места соотношение ног к Задачам равно 4 на источник, или потому, что на каждую Математическую ногу приходится 1 хвост. Разберитесь с концепцией Ставки единицы оценки: или представление измерения как отношение x к отдельной Задаче, или 1.

Задачи по математике для 6-х классов

Решение математических задач может напугать шестиклассников, но не должно. Использование нескольких простых формул и немного логики может помочь учащимся быстро вычислить ответы на кажущиеся неразрешимыми проблемы. Объясните учащимся, что вы можете узнать скорость (или скорость), с которой кто-то путешествует, если вы знаете расстояние и время, которое он проехал. И наоборот, если вы знаете скорость (скорость), с которой человек движется, а также расстояние, вы можете рассчитать время, в течение которого он путешествовал.Вы просто используете базовую формулу: скорость, умноженная на время, равна расстоянию, или r * t = d (где «*» — это символ умножения).

Приведенные ниже бесплатные распечатываемые рабочие листы связаны с такими проблемами, как эти, а также с другими важными проблемами, такими как определение наибольшего общего множителя, вычисление процентов и т. Д. Ответы для каждого рабочего листа представлены на следующем слайде сразу после каждого рабочего листа. Попросите учащихся поработать над проблемами, заполнить свои ответы в отведенных для этого пустых местах, а затем объяснить, как они могли бы прийти к решениям вопросов, по которым у них возникают трудности.Рабочие листы представляют собой отличный и простой способ выполнить быстрые формирующие оценки для всего класса математики.

Рабочий лист № 1

Распечатать PDF : Рабочий лист № 1

В этом PDF-файле ваши ученики решат такие задачи, как: «Ваш брат проехал 117 миль за 2,25 часа, чтобы вернуться домой на каникулы. Какая средняя скорость, с которой он ехал?» и «У вас есть 15 ярдов ленты для ваших подарочных коробок. Каждая коробка получает одинаковое количество ленты. Сколько ленты получит каждая из ваших 20 подарочных коробок?»

Рабочий лист №1 Решения

Решения для печати PDF : Рабочий лист № 1 Решения

Чтобы решить первое уравнение на листе, используйте основную формулу: скорость, умноженная на время = расстояние, или r * t = d . В этом случае r = неизвестная переменная, t = 2,25 часа и d = 117 миль. Выделите переменную, разделив «r» с каждой стороны уравнения, чтобы получить пересмотренную формулу: r = t ÷ d . Подставьте числа, чтобы получить: r = 117 ÷ 2,25, дает r = 52 миль в час .

Для решения второй проблемы вам даже не нужно использовать формулу — просто математика и здравый смысл. Задача заключается в простом делении: 15 ярдов ленты, разделенных на 20 ящиков, можно сократить как 15 ÷ 20 = 0,75. Таким образом, каждая коробка имеет 0,75 ярда ленты.

Рабочий лист № 2

Распечатать PDF : Рабочий лист № 2

На рабочем листе № 2 учащиеся решают задачи, требующие немного логики и знания факторов, например: «Я думаю о двух числах, 12 и еще одном числе.У 12 и другого моего числа наибольший общий делитель равен 6, а их наименьшее общее кратное — 36. О каком еще числе я думаю? »

Другие задачи требуют только базовых знаний о процентах, а также о том, как преобразовать проценты в десятичные дроби, например: «Жасмин имеет 50 шариков в сумке. 20% шариков синие. Сколько шариков синие?»

Рабочий лист № 2 Решение

Печать решений в формате PDF : Рабочий лист № 2 Решение

Для первой задачи на этом листе вам необходимо знать, что коэффициенты из 12 равны 1, 2, 3, 4, 6 и 12 ; и , кратные 12, равны 12, 24, 36 .(Вы останавливаетесь на 36, потому что проблема говорит, что это число является наименьшим общим кратным.) Давайте выберем 6 как возможное наибольшее общее кратное, потому что это наибольший множитель 12, кроме 12. кратных 6 равны 6, 12, 18 , 24, 30 и 36 . Шесть может перейти в 36 шесть раз (6 x 6), 12 может перейти в 36 три раза (12 x 3), а 18 может перейти в 36 дважды (18 x 2), а 24 — нет. Следовательно, ответ — 18, поскольку 18 — это наибольшее общее кратное, которое может перейти в 36 .

Для второго ответа решение проще: сначала преобразуйте 20% в десятичное число, чтобы получить 0.20. Затем умножьте количество шариков (50) на 0,20. Задачу можно сформулировать следующим образом: 0,20 x 50 шариков = 10 синих шариков .

31 сложная математическая загадка и задача со словами для будущих гениев

Загадка: Если есть 4 яблока и вы убираете 3, сколько у вас есть?
Ответ: Вы взяли 3 яблока, так что у вас 3!

Загадка: 300-футовый поезд, движущийся со скоростью 300 футов в минуту, должен пройти через 300-футовый туннель.Сколько времени займет поездка поезда по туннелю?
Ответ: Две минуты. У передней части поезда уходит одна минута, а остальной части поезда потребуется две минуты, чтобы очистить туннель.

Загадка: Мобильный телефон и чехол для телефона в сумме стоят 110 долларов. Сотовый телефон стоит на 100 долларов дороже чехла для телефона. Сколько стоил мобильный телефон?
Ответ: 105 долларов (не 110 долларов)

Загадка: Роберт и Дэвид сыграли несколько матчей в гольф друг против друга за неделю.На каждом матче играли за пиццу, но до конца недели пиццы не покупали. Если когда-либо Роберт и Дэвид выигрывали одинаковое количество побед, эти пиццы отменялись. Роберт выиграл четыре матча (но без пиццы), а Дэвид выиграл три пиццы. Сколько раундов было сыграно в гольф?
Ответ: Одиннадцать
Пояснение: Дэвид выиграл 7 матчей, 4 отменили 4 победы Роберта и еще 3 для выигрыша пиццы.

Загадка: Я трехзначное число.Моя вторая цифра в 4 раза больше третьей. Моя первая цифра на 3 меньше, чем вторая. Кто я?
Ответ: 141

Загадка: Я складываю пять к девяти и получаю два. Ответ правильный, но как?
Ответ: Когда будет 9 часов утра, прибавьте к нему 5 часов, и вы получите 14 часов.

Загадка: В зоопарке 100 пар собак; На каждую собаку рождаются пары младенцев. К сожалению, 23 собаки не выжили. Сколько всего собак останется?
Ответ: 977 собак
Пояснение: 100 x 2 = 200; 200 +800 = 1000; 1000-23 = 977

Загадка: Группа студентов стояла под палящим солнцем лицом к западу во время марша, прошедшего мимо.Вождь крикнул им: Поверните направо! О очередь! Налево! В конце этих команд, в каком направлении сейчас смотрят ученики?
Ответ: Восток. (Пояснение: они повернутся на 90 градусов при повороте вправо, и они повернутся на 180 градусов при повороте вокруг, и, наконец, они повернутся на 90 градусов при повороте влево. Таким образом, ученики теперь смотрят на восток.) ​​

Загадка: Половина — это треть. Что это такое?
Ответ: 1 1/2.

Загадка: Во время доставки Том может поместить 10 маленьких коробок или 8 больших коробок в картонную коробку.Всего за одну партию было отправлено 96 коробок. Маленьких ящиков было меньше, чем больших. Какое общее количество коробок он отправил?
Ответ: 11 коробок ( Пояснение: 4 маленьких коробки (4 10 = 40 коробок) + 7 больших коробок (7 8 = 56 коробок). Итак, 96 коробок и всего 11 коробок.)

Загадка: Когда Мигелю было 6 лет, его младшей сестре Лейле было полцены. Если Мигелю сегодня 40 лет, сколько лет Лейле?
Ответ: Ей 37 лет.

Загадка: Вам дано 3 положительных числа. Вы можете сложить эти числа и умножить их вместе. Результат будет таким же. Какие числа?
Ответ: 1, 2 и 3

Загадка: Если полторы курицы откладывают полтора яйца за полтора дня, сколько яиц откладывает полдюжины куриц за полдюжины дней? Ответ: 2 дюжины.

Загадка: Что можно поставить между 7 и 8, чтобы результат был больше семи, но меньше восьмерки?
Ответ: Десятичное число.7,8 больше 7, но меньше 8

Загадка: Тома попросили нарисовать числа за пределами 100 квартир, что означает, что ему придется раскрасить числа от 1 до 100. Можете ли вы подсчитать, сколько раз ему придется раскрасить число 8?
Ответ: 20 раз (8, 18, 28, 38, 48, 58, 68, 78, 80, 81, 82, 83, 84, 85, 86, 87, 88, 89, 98)

Загадка: Какое максимальное количество раз можно вычесть число 5 из 25?
Ответ: Только один раз.Это потому, что когда вы вычитаете 5 в первый раз, получается число 20, затем 15 и так далее.

Загадка: Что весит больше 16 унций соды или фунта чистого золота?
Ответ: Ни то, ни другое. Оба они весят одинаково!

Загадка: Леон работает в аквариуме. Когда он пытается поместить каждую черепаху в отдельный резервуар, у него на одну черепаху слишком много. Но если он ставит две черепахи на танк, у него слишком много на танке. Сколько черепах и сколько танков у Леона?
Ответ: У него 3 танка и 4 черепахи.

Загадка: Общая стоимость пары туфель и толстовки составляет 150 долларов. Толстовка с капюшоном стоит на 100 долларов больше, чем пара обуви. Сколько стоит каждый предмет?
Ответ: Толстовка с капюшоном стоит 125 долларов, туфли — 25 долларов

Загадка: У вас есть две монеты США общей стоимостью 0,30 доллара. Один из них не никель. Какие две монеты?
Ответ: Один — четверть, а другой — никель.

Загадка: Яйца 0 долларов.12 дюжина. Сколько яиц можно получить за доллар?
Ответ: 100 яиц по пенни каждое.

Загадка: Утка — 9 долларов, паук — 36 долларов, пчела — 27 долларов. На основании этой информации, сколько денег будет отдано кошке?
Ответ: 18 долларов (4,50 доллара за ногу)

Загадка: «Сколько стоит этот мешок картошки?» спросил мужчина. «32 фунта, разделенные на половину собственного веса», — сказал бакалейщик. Сколько весила картошка?
Ответ: 8 фунтов.

Загадка: Я трехзначное число. Моя цифра десятков на шесть больше, чем моя цифра единиц. Моя цифра сотен на восемь меньше, чем моя цифра десятков. Какой я номер?
Ответ: 193

Загадка: Мужчина вдвое старше своей младшей сестры. Он тоже вдвое младше своего отца. Через 50 лет возраст его сестры станет вдвое меньше возраста их отца. Какого возраста сейчас мужчина?
Ответ: 50 лет.

Загадка: Как сложить восемь четверок, чтобы в сумме получилось 500?
Ответ: 444 + 44 + 4 + 4 + 4 = 500

Загадка: Если семь человек встретятся друг с другом и пожмут друг другу руки только один раз, сколько рукопожатий будет?
Ответ: 21

Загадка: Если четыре человека могут построить четыре стола за четыре часа, сколько столов восемь человек могут построить за восемь часов?
Ответ : 16 таблиц

Загадка: Когда Лизе было 6 лет, ее сестра Люси была вдвое моложе.Если Люси сегодня 40 лет, сколько лет Люси?
Ответ: 37
Пояснение: Люси на 3 года младше Лизы.

Загадка: Если вы купите петуха для откладывания яиц и рассчитываете получать по три яйца каждый день на завтрак, сколько яиц у вас будет через три недели?
Ответ: Нет. Петухи яиц не откладывают.

Загадка: У фермера на земле 19 овец. Однажды обрушивается сильный шторм, и все, кроме семи, разбегаются.Сколько овец осталось у фермера?
Ответ: Семь. Все, кроме семи, сбежали.

задач 6-го класса

задач 6-го класса



1)

    Дядя Генри ехал в Галифакс, когда заметил большую лужайку.
    горилла на обочине дороги. Он с криком остановился, выпрыгнул из
    его машина. Он увидел цифру на горилле. Он не мог совсем
    видел номер, но он знал, что это четырехзначное число.И:

      1) Он вспомнил, что видел число 1.

      2) Вместо сотни он помнит, что число в 3 раза больше
      число в разряде тысяч.

      3) Он сказал, что число на одном месте в 4 раза больше числа на
      десятку.

      4) Наконец, он сказал, что число 2 стоит на месте тысячи.

    Какой номер?

    Нужна подсказка?

    Ответ


2)

    Элмер Фадд решил вырастить сад, чтобы делать салат.Он хочет
    сделать его длиной 10,1 м и шириной 4,2 м. Однако во избежание ошибок
    Банни, чтобы войти в свой сад, он должен сделать забор вокруг сада.
    Он решает сделать забор длиной 11,2 м и шириной 5,0 м. Какая площадь
    между забором и садом?

Нужна подсказка?

Ответ


3)

    Дженни купила 7 футболок, по одной каждому из семи своих братьев, для
    9 долларов.95 каждый. Кассир взимает с нее дополнительный налог с продаж в размере 13,07 доллара.
    Она вышла из магазина с скромными 7,28 доллара. Сколько денег начинала Дженни
    с?

Нужна подсказка?

Ответ


4)

    Дэйв отдал Шарлотте половину своего погона. Шарлотта отдала Джонни половину
    о погонах, которые она получила от Дэйва. Джонни сохранил 8 таких погонов и дал
    оставшиеся 10 — Дане. Сколько погов Дэйв устроил Шарлотте?

Нужна подсказка?

Ответ


5)

    В среднем канадцы тратят 958904 доллара в день.00 покупка видеоигр.
    Из этой суммы 767123,00 долларов потрачены на игры Nintendo. За одну неделю сколько
    тратят ли канадцы на игры Nintendo. Сколько они тратят на другое видео
    игры за неделю?

Нужна подсказка?

Ответ


6)

    Среднемесячное количество осадков за 6 месяцев составило 28,5 мм. Если бы это было
    дождь на 1 мм больше каждый месяц, что бы было в среднем? На сколько
    увеличилась бы общая сумма за шесть месяцев и на сколько
    в среднем были увеличены в месяц?

Нужна подсказка?

Ответ


7)

    Я атроном, объектив моей камеры 1.372 м в поперечнике. Мой друг
    Джордж — шпион, и у него есть объектив камеры диаметром 2,9 см. Моя тетя мари
    хирург, и у нее линза диаметром 1,5 см.

      1) На сколько метров диаметр объектива моей камеры больше, чем у Мари?

      2) На сколько метров меньше в поперечнике линза фотоаппарата Мари, чем у Джорджа?

      3) Сколько линз камеры Джорджа поместится на моей?

    Нужна подсказка?

    Ответ


8)

    Если вам задают следующие вопросы: Сколько российских рублей
    Вы можете купить за 300 долларов.00 канадский? Какая еще информация вам нужна
    чтобы решить проблему?

    Вам необходимо знать курс обмена. Сколько рублей можно получить за
    один канадский доллар?

    Итак, если вопрос теперь гласит: «Сколько российских рублей вы можете купить?
    с канадскими $ 300.00 зная обменный курс 20 рублей / канадский доллар.
    Что бы вы ответили?

Нужна подсказка?

Ответ


9)

Нужна подсказка?

Ответ


10)

    Джилл получила 3 ​​красных леденца, 2 синих и 2 желтых.Используйте несколько соотношений, чтобы описать конфеты, которые есть у Джилл.

Нужна подсказка?

Ответ


11)

    а) Мистер Мейсон попросил детей открыть свои учебники по математике.
    разворачивающиеся страницы, чьи номера страниц в сумме достигают 85. К каким страницам следует
    дети превращаются?

    Марк случайно вырвал страницу из своего учебника по математике. Мистер Мейсон спросил
    ему, какая это была страница.Марк сказал, что сумма номеров страниц на
    страниц было 127. Каковы номера страниц на страницах, которые разорвал Марк?
    из книги?

Нужна подсказка?

Ответ


12)

  ПРОДАЖА

Кельтская распродажа хлебобулочных изделий

Белый хлеб - 2 шт. По 1,25 доллара США.
Ржаной хлеб - 2 по 1,35 доллара
Луковые рулетики - 6 по 1 доллар.00
Булочки для гамбургеров - 6 за 0,85 доллара
Булочки для хот-догов - 6 за 0,69 доллара  

Мелани и Кевин идут за мамой в пекарню Celtic Bakery.
Им сказали купить 15 луковых булочек, 10 булочек для гамбургеров и 2 буханки
ржаной хлеб. Во сколько им это будет стоить?

Нужна подсказка?

Ответ


13)

    a) Если вы сэкономили 2 доллара 1 января, то 4 доллара.00:00 1 февраля, $ 6,00 в
    1 марта, 8 долларов США 1 апреля и т. Д., Сколько денег вы бы сэкономили
    один год?

    б) Если вы сэкономили 2 доллара 1 января, 4 доллара 1 февраля, 8 долларов
    1 марта, 1 апреля — 16 долларов и т. Д. Сколько денег вы бы сэкономили в
    один год?

Нужна подсказка?

Ответ


14)

    Числовая прямая от 0 до 2 разделена на семь равных отрезков.Какая фракция называет точку А? Какая фракция называет точку B?

Нужна подсказка?

Ответ


15)

    Деннис Грозный думает о двух числах. Их самое большое общее
    Фактор равен 6. Их наименьшее общее кратное — 36. Одно из чисел — 12.
    Какой другой номер?

Нужна подсказка?

Ответ


16)

Нужна подсказка?

Ответ


17)

Нужна подсказка?

Ответ


18)

    Этель составила список всех целых чисел от 1 до 100.Как
    много раз она писала цифру 2?

Нужна подсказка?

Ответ


19)

    Сколько треугольников на этом рисунке?

Нужна подсказка?

Ответ


20)

    Можете ли вы решить этот магический квадрат?

    Поместите оставшиеся числа от 0 до 15 в 16 маленьких квадратов.В
    сумма четырех чисел в каждой строке, столбце и двух диагоналях должна быть 30.

  15 | | | 12
---- + ---- + ---- + ----
| 10 | 9 |
---- + ---- + ---- + ----
| | | 11
---- + ---- + ---- + ----
3 | | | 0  

Нужна подсказка?

Ответ


21)

    Мэри и Джейсон изготавливали пропеллеры для своих деревянных вертолетов.Мэри поставила номер на пропеллер и заметила, что, когда она повернула
    пропеллер у нее был такой же номер.

    Джейсон пытался придумать число, которое он мог бы поставить на свой
    имел ту же вращательную симметрию (выглядит так же перевернутой). Что
    следующее большее число, обладающее этим свойством?

Нужна подсказка?

Ответ


22)

    а) Мистери миссис Каллахан и их трое детей заплатили 7 долларов за билеты.
    Какую игру они планируют посмотреть?
  СОФТБОЛ: Взрослые 2 доллара США Дети 1 доллар США
БАСКЕТБОЛ: Взрослые 3,00 доллара США Дети 1,50 доллара США
ФУТБОЛ: Взрослые 5,00 $ Дети 2,50 $  

б) Сколько сдачи получили бы мистер и миссис Каллахан, если бы
дети посмотрели баскетбольный матч и заплатили за это счетом в 20 долларов?

Нужна подсказка?

Ответ


23)

    Фермер Том поставил квадратный забор вокруг своего огорода, чтобы сохранить
    олень от еды его кукурузы.Одна сторона была 10 метров в длину. Если сообщения
    были размещены на расстоянии 2 м друг от друга, сколько столбов он использовал?

Нужна подсказка?

Ответ


24)

    Амми передала находившимся у нее девочкам корзину с клубникой.
    партия. Перед вечеринкой она съела 5 ягод клубники и подарила другу 3. Восемь.
    на вечеринку приехали девушки. Первая девочка взяла клубнику, вторая
    девочка взяла 3 клубники, третья девочка — 5 клубники и так далее.После того, как последняя девочка взяла клубнику, корзина опустела. Сколько
    клубника была в корзине вначале?

Нужна подсказка?

Ответ


25)

    Роб хотел пособие. Его отец дал ему выбор получить
    это еженедельно или ежедневно. Он сказал, что либо заплатит ему 1,25 доллара.
    неделю или заплатить ему за неделю следующим образом: в понедельник он
    дать ему 0 долларов.01; Во вторник 0,02 доллара США; В среду 0,04 доллара США и по воскресенье.
    Что бы вы посоветовали сделать Робу, чтобы он мог получать больше пособий?

Нужна подсказка?

Ответ


26)

    Сторона квадрата B в четыре раза больше длины стороны квадрата
    A. Во сколько раз площадь квадрата B больше площади квадрата
    А?

Нужна подсказка?

Ответ


27)

    Улитка Шейн вздрогнула.На какой стороне он будет, когда
    он прополз 13/20 расстояния вокруг правильного пятиугольника, равного
    стороны?

Нужна подсказка?

Ответ


28)

    Количество оставшихся в день часов составляло одну треть от
    количество часов уже прошло. Сколько часов осталось в дне?

Нужна подсказка?

Ответ


29)

    Прямоугольный кухонный стол в три раза больше его ширины.

    Если бы он был на 3 м короче и на 3 м шире, это был бы квадрат.

    Каковы размеры прямоугольного стола?

Нужна подсказка?

Ответ


30)

    Производитель утверждает, что новое моторное масло экономит 5 процентов
    бензин, используемый автомобилем. Если вы проезжаете 24000 километров в год, и он получает
    32 километра галлона бензина, сколько галлонов бензина может
    вы экономите за 1 год?

Нужна подсказка?

Ответ


31)

    Какой начальный номер (?)?

Нужна подсказка?

Ответ


32)

    Ковровое покрытие продается и Mrs.Дойл ищет ковер для
    ее гостиная. Ее гостиная 4 м на 5 м. Сколько это будет ей стоить
    сделать это по продажной цене?
 
ПРОДАЖА КОВРОВ
Обычный квадратный метр стоимостью 9,99 долларов США.

Сейчас в продаже для

Скидка 20%  

Нужна подсказка?

Ответ


33)

    Дерек завел машину (автомат) проехал 9 км и проехал 3 минуты
    ожидание на светофоре.Примерно сколько бензина использовала машина Дерека?

    ДАННАЯ ДИАГРАММА:

  ИСПОЛЬЗОВАНИЕ АВТОМОБИЛЬНОГО ТОПЛИВА


Начальный 0,015 литра


На холостом ходу в течение 1 минуты 0,047 литра


Движущийся


Ручное переключение передач 30 км 3,0 литра


Автоматическое переключение передач 27 км 3,0 литра  

Нужна подсказка?

Ответ


34)

    Паром Pictou Ferryboat заполнен, когда на борту десять автомобилей.это
    также полный, когда на борту шесть грузовиков. Паром никогда не везет
    легковые и грузовые автомобили одновременно.

    Паром совершил пять переходов через реку и на каждом был полон.
    поездка. Он перевез через реку 42 легковых и грузовых автомобиля.

    Сколько машин всего перевез паром за пять рейсов?

Нужна подсказка?

Ответ


35)

    Джереми столкнулся с этой проблемой на первом экзамене по математике.Он
    возникли большие проблемы с ответом на вопрос. Заменить каждую букву
    с другим номером. Идентичные буквы следует заменить на
    такое же количество.
  SPART
х 4
----------
ЛОВУШКИ  

Можете ли вы ему помочь?
Нужна подсказка?

Ответ


36)

    В доме Порки было 100 м 2 жилой площади.Затем он добавил
    Комната была 4м на 5м. Какое было дробное увеличение жилплощади?
    Какой был процент увеличения жилплощади?

Нужна подсказка?

Ответ


37)

    Шери, Бет и Хасинта — дочери г-на Салливана, г-на Маршана
    и г-н Бенуа. Четверо из этих людей играют в парный теннис. Г-на Бенуа
    дочь и мистер Салливан — партнеры.Отец Шери и мистер Маршан
    дочь тоже партнеры. Никаких комбинаций отца и дочери не существует.

    Кто отец Чери?

Нужна подсказка?

Ответ


38)

    Маг сказал: «Среднее из семи чисел равно 49. Если 1 — это
    прибавляется к первому числу, ко второму числу прибавляется 2, прибавляется 3
    до третьего числа и так до седьмого числа », что нового
    средний?

Нужна подсказка?

Ответ


39)

    Стоимость проезда на автобусе составила 2 доллара.10. Какое расстояние было пройдено, когда
    стоимость проезда составляет 0,90 доллара США за первую 1/3 км и 0,10 доллара США за каждую дополнительную 1/8 километра.
    километр?

Нужна подсказка?

Ответ


40)

    Вот вам вопрос !!!

    Одна из этих вещей здесь не принадлежит, одна из этих вещей
    не то же самое. Вы можете сказать, какой куб?

Нужна подсказка?

Ответ


41)

    Наибольший общий делитель двух чисел равен 30.Их наименее распространенные
    кратное 420. Одно из чисел — 210. Вы можете найти другое число?

Нужна подсказка?

Ответ


42)

    Кожура банана весит около 1/8 от общего веса
    банан. Если вы купите 3 кг бананов по 1 кг за 0,60 доллара, примерно сколько стоит
    вы платите за кожуру банана? Для самого банана? Округлить до ближайшего
    цент

Нужна подсказка?

Ответ


43)

    Два плотника решили спроектировать парты для учеников младшей школы.
    Высокая.Размеры стола указаны на рисунке. Сколько бы дерева они
    нужно 30 столов? (в см 2 ).

Нужна подсказка?

Ответ


44)

    Болтун сделал следующие фигурки из куска дерева, 12 гвоздей.
    и резинка. Какова площадь рисунка (в единицах 2 )?
    Объясните свой метод:

Нужна подсказка?

Ответ


45)

    У этих параллелограммов одинаковая площадь? Откуда вы знаете?

Нужна подсказка?

Ответ


15 самых сложных вопросов по SAT математике

Хотите проверить себя, отвечая на самые сложные вопросы по математике SAT? Хотите знать, что делает эти вопросы такими сложными и как их лучше всего решать? Если вы готовы по-настоящему погрузиться в математический раздел SAT и нацелиться на этот высший балл, то это руководство для вас.

Мы собрали то, что мы считаем , из 15 самых сложных вопросов для текущего SAT , со стратегиями и ответами на каждый из них. Все это сложные вопросы SAT Math из практических тестов SAT College Board, а это значит, что их понимание — один из лучших способов учиться для тех из вас, кто стремится к совершенству.

Изображение: Соня Севилья / Викимедиа

Краткий обзор SAT Math

Третий и четвертый разделы SAT всегда будут математическими разделами .Первый математический подраздел (обозначенный «3») не позволяет использовать , , а не , а второй математический подраздел (обозначенный как «4») разрешает использование калькулятора. Однако не беспокойтесь о разделе без калькулятора: если вам не разрешено использовать калькулятор для ответа на вопрос, это означает, что вам не нужен калькулятор, чтобы ответить на него.

Каждый математический подраздел расположен в порядке возрастания сложности (где чем больше времени требуется на решение задачи и чем меньше людей ответят на нее правильно, тем сложнее).В каждом подразделе вопрос 1 будет «легким», а вопрос 15 — «сложным». Однако возрастающая сложность сбрасывается с простого на сложный на сетке.

Таким образом, вопросы с несколькими вариантами ответов расположены по возрастающей сложности (вопросы 1 и 2 будут самыми легкими, вопросы 14 и 15 будут самыми сложными), но уровень сложности сбрасывается для секции сетки (то есть вопросы 16 и 17 снова будут будьте «легкими», и вопросы 19 и 20 будут очень сложными).

Таким образом, за очень немногими исключениями, наиболее сложные математические задачи SAT будут сгруппированы в конце сегментов с несколькими вариантами ответов или во второй половине вопросов сетки. Однако, помимо места в тесте, у этих вопросов есть еще несколько общих черт. Через минуту мы рассмотрим примеры вопросов и способы их решения, а затем проанализируем их, чтобы выяснить, что общего у этих типов вопросов.

Но сначала: стоит ли вам прямо сейчас сосредоточиться на самых сложных математических вопросах?

Если вы только начинаете подготовку к учебе (или если вы просто пропустили этот первый, важный шаг), обязательно остановитесь и пройдите полный практический тест, чтобы определить свой текущий уровень оценок. Ознакомьтесь с нашим руководством по всем бесплатным практическим тестам SAT, доступным в Интернете, а затем сядьте и сдавайте все сразу.

Самый лучший способ оценить свой текущий уровень — это просто пройти практический тест SAT, как если бы он был настоящим, соблюдая строгий график и работая без перерывов только с разрешенными перерывами (мы знаем — вероятно, не ваш любимый способ провести субботу) . Как только вы получите хорошее представление о своем текущем уровне и процентильном рейтинге, вы можете установить контрольные точки и цели для получения окончательного результата по SAT Math.

Если вы в настоящее время набираете баллы в диапазоне 200–400 или 400–600 по SAT Math, лучше всего сначала ознакомиться с нашим руководством по повышению своего балла по математике , чтобы он постоянно был на уровне 600 или выше, прежде чем начать. в попытке решить самые сложные математические задачи на тесте.

Если, однако, вы уже набрали больше 600 баллов по математике и хотите проверить свои способности на реальном SAT, то обязательно переходите к остальной части этого руководства. Если вы стремитесь к совершенству (или близкому к нему), вам необходимо знать, как выглядят самые сложные вопросы по математике SAT и как их решать.К счастью, именно этим мы и займемся.

ПРЕДУПРЕЖДЕНИЕ: Поскольку количество официальных практических тестов SAT ограничено, вы можете подождать, чтобы прочитать эту статью, пока не попробуете все или большую часть первых четырех официальных практических тестов (так как большинство вопросов, приведенных ниже, были приняты. из этих тестов). Если вы беспокоитесь о том, чтобы испортить эти тесты, прекратите читать это руководство сейчас; вернитесь и прочтите, когда вы их закончите.

Теперь перейдем к нашему списку вопросов (ууу)!

Изображение: Niytx / DeviantArt

15 самых сложных вопросов по SAT-математике

Теперь, когда вы уверены, что должны попытаться ответить на эти вопросы, давайте приступим прямо к делу! Мы собрали 15 самых сложных вопросов по SAT Math, которые вы можете попробовать ниже, а также пошаговые инструкции, как получить ответ (если вы в тупике).

Нет калькулятора Вопросы по SAT по математике

Вопрос 1

$$ C = 5/9 (F-32) $$

Приведенное выше уравнение показывает, как температура $ F $, измеренная в градусах Фаренгейта, соотносится с температурой $ C $, измеренной в градусах Цельсия. Основываясь на уравнении, какое из следующих утверждений должно быть верным?

  1. Повышение температуры на 1 градус по Фаренгейту эквивалентно повышению температуры на 5/9 градусов Цельсия.
  2. Повышение температуры на 1 градус Цельсия эквивалентно повышению температуры на 1 градус.8 градусов по Фаренгейту.
  3. Повышение температуры на 5 долларов / 9 градусов по Фаренгейту эквивалентно повышению температуры на 1 градус Цельсия.

A) только I
B) только II
C) только III
D) только I и II

ОБЪЯСНЕНИЕ ОТВЕТА: Думайте об уравнении как об уравнении для линии

$$ y = mx + b $$

, где в данном случае

$$ C = {5} / {9} (F − 32) $$

или

$$ C = {5} / {9} F — {5} / {9} (32) $$

Вы можете видеть, что наклон графика составляет $ {5} / {9} $, что означает, что при увеличении на 1 градус по Фаренгейту увеличение составляет $ {5} / {9} $ на 1 градус Цельсия.

$$ C = {5} / {9} (F) $$

$$ C = {5} / {9} (1) = {5} / {9} $$

Следовательно, утверждение I верно. Это эквивалентно тому, что увеличение на 1 градус Цельсия равно увеличению на $ {9} / {5} $ градусов по Фаренгейту.

$$ C = {5} / {9} (F) $$

$$ 1 = {5} / {9} (F) $$

$$ (F) = {9} / {5} $$

Поскольку $ {9} / {5} $ = 1.8, утверждение II верно.

Единственный ответ, в котором и утверждение I, и утверждение II являются истинными, — это D , но если у вас есть время и вы хотите быть абсолютно внимательными, вы также можете проверить, соответствует ли утверждение III (увеличение на $ {5} / { 9} $ градус Фаренгейта равен увеличению температуры на 1 градус Цельсия) верно:

$$ C = {5} / {9} (F) $$

$$ C = {5} / {9} ({5} / {9}) $$

$$ C = {25} / {81} (\ which \ is ≠ 1) $$

Увеличение на 5 долларов / 9 градусов по Фаренгейту приводит к увеличению на {25} / {81} долларов, а не на 1 градус Цельсия, и поэтому утверждение III неверно. 2 $
D) Значение не может быть определено на основе предоставленной информации.12 $$

Окончательный ответ: A.

Вопрос 4

Точки A и B лежат на окружности радиуса 1, а длина дуги $ {AB} ↖⌢ $ равна $ π / 3 $. Какая часть окружности окружности равна длине дуги $ {AB} ↖⌢ $?

ОБЪЯСНЕНИЕ ОТВЕТА: Чтобы выяснить ответ на этот вопрос, вам сначала нужно знать формулу для определения длины окружности.

Длина окружности $ C $ равна $ C = 2πr $, где $ r $ — радиус окружности.Для данной окружности радиусом 1 длина окружности равна $ C = 2 (π) (1) $ или $ C = 2π $.

Чтобы узнать, какая часть окружности составляет длину $ {AB} ↖⌢ $, разделите длину дуги на длину окружности, что даст $ π / 3 ÷ 2π $. Это деление можно представить как $ π / 3 * {1/2} π = 1/6 $.

Дробь $ 1/6 $ также может быть переписана как $ 0,166 $ или 0,167 $.

Окончательный ответ: 1/6 доллара, 0,166 доллара или 0,167 доллара.

Вопрос 5

$$ {8-i} / {3-2i} $$

Если приведенное выше выражение переписать в форме $ a + bi $, где $ a $ и $ b $ — действительные числа, каково значение $ a $? (Примечание: $ i = √ {-1} $)

ОБЪЯСНЕНИЕ ОТВЕТА: Чтобы переписать $ {8-i} / {3-2i} $ в стандартной форме $ a + bi $, вам нужно умножить числитель и знаменатель $ {8-i} / {3- 2i} $ сопряженным, $ 3 + 2i $.2 = -1 $, последняя дробь может быть уменьшена упрощенно до

$$ {24 + 16i-3i + 2} / {9 — (- 4)} = {26 + 13i} / {13} $$

, что упрощается до 2 + i $. Следовательно, когда $ {8-i} / {3-2i} $ переписывается в стандартной форме a + bi, значение a равно 2.

Окончательный ответ: A.

Вопрос 6

В треугольнике $ ABC $ мера $ ∠B $ равна 90 °, $ BC = 16 $ и $ AC $ = 20. Треугольник $ DEF $ похож на треугольник $ ABC $, где вершины $ D $, $ E $ и $ F $ соответствуют вершинам $ A $, $ B $ и $ C $ соответственно, а также каждой стороне треугольника $. DEF $ составляет $ 1/3 $ длины соответствующей стороны треугольника $ ABC $.2} = √ {400-256} = √ {144} = 12 $$

Поскольку треугольник DEF подобен треугольнику ABC, с вершиной F, соответствующей вершине C, мера $ \ angle ∠ {F} $ равна мере $ \ angle ∠ {C} $. Следовательно, $ sin F = sin C $. От сторон треугольника ABC,

$$ sinF = {\ Against \ side} / {\ hypotenuse} = {AB} / {AC} = {12} / {20} = {3} / {5} $$

Следовательно, $ sinF = {3} / {5} $.

Окончательный ответ: {3} / {5} $ или 0,6.

Вопросы SAT по математике, разрешенные калькулятором

Вопрос 7

Неполная таблица выше суммирует количество учащихся-левшей и учащихся-правшей с разбивкой по полу для учащихся восьмых классов средней школы им. Кейзеля.Учениц-правшей в 5 раз больше, чем учениц-левшей, и учеников-правшей в 9 раз больше, чем учащихся-левшей. Если в школе 18 учеников-левшей и 122 учащихся-правшей, что из следующего наиболее близко к вероятности того, что случайно выбранный ученик-правша будет женщиной? (Примечание: предположим, что ни один из восьмиклассников не является одновременно правшой и левшой.)

А) 0.410
B) 0,357
C) 0,333
D) 0,250

ОБЪЯСНЕНИЕ ОТВЕТА: Чтобы решить эту проблему, вы должны создать два уравнения, используя две переменные ($ x $ и $ y $) и предоставленную вам информацию. Пусть $ x $ будет количеством учениц-левшей и пусть $ y $ будет количеством учениц-левшей. Используя информацию, приведенную в задаче, количество учащихся-правшей будет составлять 5 долларов США, а количество учащихся-правшей будет составлять 9 лет.Поскольку общее количество студентов-левшей составляет 18, а общее количество студентов-правшей — 122, система уравнений ниже должна быть верной:

$$ x + y = 18 $$

$$ 5x + 9y = 122 $$

Когда вы решаете эту систему уравнений, вы получаете $ x = 10 $ и $ y = 8 $. Таким образом, из 122 учащихся-правшей 5 * 10, или 50, — девушки. Следовательно, вероятность того, что случайным образом выбранный студент-правша будет женщиной, составляет {50} / {122} $, что с точностью до тысячных составляет 0,410.

Окончательный ответ — А.

Вопросы 8 и 9

Используйте следующую информацию как для вопроса 7, так и для вопроса 8.

Если покупатели входят в магазин со средней скоростью $ r $ покупателей в минуту и ​​каждый остается в магазине в течение среднего времени T $ минут, среднее количество покупателей в магазине, N $, в любой момент времени равно задается формулой $ N = rT $. Эта связь известна как закон Литтла.

По оценкам владельца магазина Good Deals Store, в рабочее время в магазин заходит в среднем 3 покупателя в минуту, и каждый из них остается в среднем на 15 минут.Владелец магазина использует закон Литтла, чтобы оценить, что в магазине одновременно находится 45 покупателей.

Вопрос 8

Закон Литтла может применяться к любой части магазина, например к определенному отделу или кассовым линиям. Владелец магазина определяет, что в рабочее время примерно 84 покупателя в час совершают покупку, и каждый из этих покупателей проводит в очереди в кассе в среднем 5 минут. Сколько в среднем покупателей в любое время в рабочее время ожидают в очереди у кассы, чтобы совершить покупку в магазине Good Deals Store?

ОБЪЯСНЕНИЕ ОТВЕТА: Поскольку в вопросе говорится, что закон Литтла может применяться к любой отдельной части магазина (например, только к кассе), тогда среднее количество покупателей, $ N $, в очереди к кассе в любой time равно $ N = rT $, где $ r $ — это количество покупателей, заходящих в кассу в минуту, а $ T $ — это среднее количество минут, которое каждый покупатель проводит в очереди.

Поскольку 84 покупателя в час совершают покупку, 84 покупателя в час входят в кассу. Однако это необходимо преобразовать в количество покупателей в минуту (для использования с $ T = 5 $). Поскольку в часе 60 минут, тариф составляет $ {84 \ shoppers \ per \ hour} / {60 \ minutes} = 1,4 $ покупателя в минуту. Используя данную формулу с $ r = 1,4 $ и $ T = 5 $, получаем

$$ N = rt = (1.4) (5) = 7 $$

Таким образом, среднее количество покупателей, $ N $, в очереди на кассу в любое время в рабочее время равно 7.

Окончательный ответ 7.

Вопрос 9

Владелец магазина Good Deals Store открывает новый магазин в другом конце города. По оценкам владельца нового магазина, в рабочее время в него заходят в среднем 90 покупателей в час, и каждый из них остается в среднем на 12 минут. Среднее количество покупателей в новом магазине в любой момент времени на какой процент меньше среднего количества покупателей в исходном магазине в любое время? (Примечание: игнорируйте символ процента при вводе ответа.Например, если ответ 42,1%, введите 42,1)

ОБЪЯСНЕНИЕ ОТВЕТА: Согласно исходной информации, расчетное среднее количество покупателей в исходном магазине в любое время (N) составляет 45. В вопросе говорится, что в новом магазине менеджер оценивает, что в среднем 90 покупателей в час (60 минут) заходят в магазин, что эквивалентно 1,5 покупателям в минуту (r). Менеджер также подсчитал, что каждый покупатель остается в магазине в среднем 12 минут (T).Таким образом, по закону Литтла в каждый момент времени в новом магазине в среднем находится $ N = rT = (1.5) (12) = 18 $ покупателей. Это

$$ {45-18} / {45} * 100 = 60 $$

На

% меньше, чем среднее количество покупателей в исходном магазине в любое время.

Окончательный ответ — 60.

Вопрос 10

На плоскости $ xy $ точка $ (p, r) $ лежит на прямой с уравнением $ y = x + b $, где $ b $ — константа. Точка с координатами $ (2p, 5r) $ лежит на прямой с уравнением $ y = 2x + b $.Если $ p ≠ 0 $, каково значение $ r / p $?

A) 2/5 долларов США

B) 3/4 $

C) 4/3 долл. США

D) $ 5/2 $

ОБЪЯСНЕНИЕ ОТВЕТА: Поскольку точка $ (p, r) $ лежит на прямой с уравнением $ y = x + b $, точка должна удовлетворять уравнению. Подстановка $ p $ вместо $ x $ и $ r $ вместо $ y $ в уравнение $ y = x + b $ дает $ r = p + b $, или $ \ bi b $ = $ \ bi r- \ bi p $.

Аналогично, поскольку точка $ (2p, 5r) $ лежит на прямой с уравнением $ y = 2x + b $, точка должна удовлетворять уравнению.Замена $ 2p $ на $ x $ и $ 5r $ на $ y $ в уравнении $ y = 2x + b $ дает:

$ 5r = 2 (2p) + b $

$ 5r = 4p + b $

$ \ bi b $ = $ \ bo 5 \ bi r- \ bo 4 \ bi p $.

Затем мы можем установить два уравнения, равных $ b $, равным друг другу и упростить:

$ б = р-п = 5р-4п $

$ 3p = 4r $

Наконец, чтобы найти $ r / p $, нам нужно разделить обе части уравнения на $ p $ и на $ 4 $:

$ 3p = 4r $

3 доллара США = {4r} /

доллара США на человека

$ 3/4 = р / п $

Правильный ответ: B , 3/4 доллара.

Если вы выбрали варианты A и D, возможно, вы неправильно сформировали свой ответ из коэффициентов в пункте $ (2p, 5r) $. Если вы выбрали вариант C, возможно, вы перепутали $ r $ и $ p $.

Обратите внимание, что пока это находится в разделе калькулятора теста SAT, вам совершенно не нужен калькулятор для его решения!

Вопрос 11

Зерновой бункер состоит из двух правых круглых конусов и правого круглого цилиндра с внутренними размерами, представленными на рисунке выше. 2h $$

можно использовать для определения общего объема силоса.2) (5) = ({4} / {3}) (250) π $$

, что примерно равно 1047,2 кубических футов.

Окончательный ответ — D.

Вопрос 12

Если $ x $ — это среднее (среднее арифметическое) для $ m $ и $ 9 $, $ y $ — это среднее значение для $ 2m $ и $ 15 $, а $ z $ — это среднее значение для $ 3m $ и $ 18 $, что есть среднее значение $ x $, $ y $ и $ z $ в пересчете на $ m $?

A) $ m + 6 $ 90 272 B) $ m + 7 $ 90 272 C) $ 2m + 14 $ 90 272 D) $ 3m + 21 $

ОБЪЯСНЕНИЕ ОТВЕТА: Поскольку среднее (среднее арифметическое) двух чисел равно сумме двух чисел, деленных на 2, уравнения $ x = {m + 9} / {2} $, $ y = {2m +15} / {2} $, $ z = {3m + 18} / {2} $ верны.2-x- {11} / {4} $$

и

$$ y = k $$

Реальное решение системы двух уравнений соответствует точке пересечения графиков этих двух уравнений на плоскости $ xy $.

График $ y = k $ — это горизонтальная линия, которая содержит точку $ (0, k) $ и трижды пересекает график кубического уравнения (поскольку оно имеет три действительных решения). Учитывая график, единственная горизонтальная линия, которая трижды пересекала бы кубическое уравнение, — это линия с уравнением $ y = −3 $ или $ f (x) = −3 $.2 $$

Динамическое давление $ q $, создаваемое жидкостью, движущейся со скоростью $ v $, можно найти с помощью приведенной выше формулы, где $ n $ — постоянная плотность жидкости. Инженер-авиастроитель использует формулу для определения динамического давления жидкости, движущейся со скоростью $ v $, и той же жидкости, движущейся со скоростью 1,5 $ v $. Каково отношение динамического давления более быстрой жидкости к динамическому давлению более медленной жидкости?

ОБЪЯСНЕНИЕ ОТВЕТА: Чтобы решить эту проблему, вам необходимо задать уравнения с переменными.2 = (2.25) q_1 $$

Следовательно, коэффициент динамического давления более быстрой жидкости равен

$$ {q2} / {q1} = {2.25 q_1} / {q_1} = 2.25 $$

Окончательный ответ — 2,25 или 9/4.

Вопрос 15

Для полинома $ p (x) $ значение $ p (3) $ равно $ -2 $. Что из следующего должно быть верным относительно $ p (x) $?

A) $ x-5 $ — множитель $ p (x) $.
B) $ x-2 $ является множителем $ p (x) $.
C) $ x + 2 $ является множителем $ p (x) $.
D) Остаток от деления $ p (x) $ на $ x-3 $ равен $ -2 $.1 $ и не выше), остаток — действительное число.

Следовательно, $ p (x) $ можно переписать как $ p (x) = (x + k) q (x) + r $, где $ r $ — действительное число.

В вопросе указано, что $ p (3) = -2 $, поэтому должно быть верно, что

$$ — 2 = p (3) = (3 + k) q (3) + r $$

Теперь мы можем ввести все возможные ответы. Если ответ A, B или C, $ r $ будет $ 0 $, а если ответ D, $ r $ будет $ -2 $.

A. $ -2 = p (3) = (3 + (-5)) q (3) + 0 $ 90 272 $ -2 = (3-5) q (3) $ 90 272 $ -2 = (- 2 ) q (3) $

Это могло быть правдой, но только если $ q (3) = 1 $

Б.$ -2 = p (3) = (3 + (-2)) q (3) + 0 $
$ -2 = (3-2) q (3) $
$ -2 = (-1) q ( 3) $

Это могло быть правдой, но только если $ q (3) = 2 $

C. $ -2 = p (3) = (3 + 2) q (3) + 0 $ 90 272 $ -2 = (5) q (3) $

Это может быть правдой, но только если $ q (3) = {- 2} / {5} $

D. $ -2 = p (3) = (3 + (-3)) q (3) + (-2) $ 90 272 $ -2 = (3 — 3) q (3) + (-2) $
-2 = (0) q (3) + (-2) 9000 3 долл. США

Это всегда будет истинным независимо от того, что такое $ q (3) $.

Из вариантов ответа единственное, что должно быть истинным относительно $ p (x) $, — это D, а остаток от деления $ p (x) $ на $ x-3 $ равен -2.

Окончательный ответ — D.

Хотите улучшить свой SAT на 160 баллов? Мы написали руководство о 5 лучших стратегиях, которые вы должны использовать, чтобы улучшить свой результат. Скачать бесплатно сейчас:

Вы заслуживаете того, чтобы вздремнуть, задав эти вопросы.

Что общего у самых сложных вопросов по SAT Math?

Важно понимать, что делает эти сложные вопросы «сложными». Поступая таким образом, вы сможете понять и решить похожие вопросы, когда вы увидите их в день тестирования, а также получите лучшую стратегию для выявления и исправления ваших предыдущих математических ошибок SAT.

В этом разделе мы рассмотрим, что общего у этих вопросов, и приведем примеры каждого типа.Некоторые из причин, по которым самые сложные вопросы по математике являются самыми сложными вопросами по математике, заключаются в том, что они:

# 1: Проверить несколько математических понятий одновременно

Здесь мы должны иметь дело с мнимыми числами и дробями одновременно.

Секрет успеха: Подумайте, какую применимую математику вы могли бы использовать для решения задачи, выполняйте по одному шагу за раз и пробуйте каждый метод, пока не найдете тот, который работает!

# 2: задействовать множество шагов

Помните: чем больше шагов вам нужно предпринять, тем легче где-то напортачить!

Мы должны решить эту проблему поэтапно (делая несколько средних), чтобы разблокировать остальные ответы в эффекте домино.Это может сбивать с толку, особенно если вы в стрессе или у вас не хватает времени.

Секрет успеха: Не торопитесь, делайте шаг за шагом и перепроверяйте свою работу, чтобы не ошибиться!

# 3: Проверьте концепции, с которыми вы мало знакомы

Например, многие учащиеся менее знакомы с функциями, чем с дробями и процентами, поэтому большинство функциональных вопросов считаются задачами «высокой сложности».

Если вы не разбираетесь в функциях, это может быть сложной проблемой.

Секрет успеха: Просмотрите математические концепции, с которыми вы не так хорошо знакомы, например, функции. Мы предлагаем использовать наши отличные бесплатные руководства по тестированию SAT Math.

# 4: написаны необычно или запутанно

Может быть сложно точно выяснить, какие вопросы задает , не говоря уже о том, как их решить. Это особенно актуально, когда вопрос находится в конце раздела, а у вас не хватает времени.

Поскольку в этом вопросе содержится так много информации без диаграммы, его может быть сложно решить за ограниченное время.

Секрет успеха: Не торопитесь, проанализируйте, что от вас просят, и нарисуйте диаграмму, если это вам поможет.

# 5: Используйте много разных переменных

При таком большом количестве различных переменных очень легко запутаться.

Секрет успеха: Не торопитесь, проанализируйте то, что от вас просят, и подумайте, является ли включение цифр хорошей стратегией для решения проблемы (это не относится к вопросу выше, но может быть ко многим другим. SAT переменные вопросы).

Итоги

SAT — это марафон, и чем лучше вы к нему подготовитесь, тем лучше вы будете себя чувствовать в день теста. Знание того, как отвечать на самые сложные вопросы, которые может бросить вам тест, сделает сдачу настоящего SAT намного менее сложной задачей.

Если вы считаете, что эти вопросы были легкими, не стоит недооценивать влияние адреналина и усталости на вашу способность решать проблемы. Продолжая учиться, всегда придерживайтесь надлежащих рекомендаций по времени и старайтесь по возможности проходить полные тесты. Это лучший способ воссоздать реальную среду тестирования, чтобы вы могли подготовиться к реальной сделке.

Если вы считаете, что эти вопросы были сложными, обязательно укрепит свои математические знания, просмотрев наши индивидуальные руководства по математическим темам для SAT. Здесь вы увидите более подробные объяснения рассматриваемых тем, а также более подробную разбивку ответов.

Что дальше?

Чувствовали, что эти вопросы оказались сложнее, чем вы ожидали? Взгляните на все темы, затронутые в разделе SAT по математике, а затем отметьте, какие разделы были для вас особенно трудными.Затем взгляните на наши индивидуальные руководства по математике, которые помогут вам укрепить любую из этих слабых сторон.

Не хватает времени на сдачу экзамена по математике? Наш гид поможет вам выиграть время и увеличить свой счет.

Хотите набрать наивысший балл? Ознакомьтесь с нашим руководством о том, как получить идеальные 800 баллов по математике в разделе SAT, написанном отличником.

Хотите улучшить свой SAT на 160 баллов?

Посетите наши лучшие в своем классе онлайн-классы подготовки к SAT.Мы гарантируем возврат ваших денег , если вы не улучшите свой SAT на 160 баллов или более.

Наши классы полностью онлайн, и их ведут эксперты SAT. Если вам понравилась эта статья, вам понравятся наши классы. Наряду с занятиями под руководством экспертов вы получите индивидуальное домашнее задание с тысячами практических задач, организованных по индивидуальным навыкам, чтобы вы учились наиболее эффективно. Мы также дадим вам пошаговую индивидуальную программу, которой вы будете следовать, чтобы вы никогда не запутались, что изучать дальше.

Добавить комментарий

Ваш адрес email не будет опубликован. Обязательные поля помечены *